Вы находитесь на странице: 1из 93

Item: 1 of 24 ~ 1 • Mark -<] C> Jill ~· ~J

QID: 237 16 ..1 Previous Next Lab'V!I!ues Notes Calculator

•1 •
A girl is born vaginally at 31 weeks' gestation to a 19-year-old woman who did not receive any prenatal care . The baby ~~AI
•2
has Apgar scores of 9 and 10 at 1 and 5 minutes, respectively, and does well in itially, but on day 3 she develops a
•3 continuous machine-li ke heart murmur heard best at the left sternal border. She also becomes tachypneic with a widened
•4 pulse pressure . She remains acyanotic and does not exh ibit increased oxygen requirement .
•5
•6 Which of the following is the most effective management?'
•7
•8 A. Administer indomethacin
•9 B. Administer prostag landin E1
• 10
C. Aortic va lve replacement
• 11

• 12 D. Close monitoring
• 13 E. Surgica l correction with arteria l or atrial switch
• 14
• 15
• 16
• 17
• 18
• 19
• 20
• 21
• 22
• 23
• 24

Lock
s
Suspend
0
End Block
Item: 1 of 24 ~ 1 • Mark -<] C> Jill ~· ~J
QID: 237 16 ..1 Previous Next Lab'V!I!ues Notes Calculator

1
•2
The correct answer is A. 710/o chose this .
•3
This patient has patent ductus arteriosus (PDA), as indicated by the wide pulse pressure, continuous machinery murmur, and
•4
normal oxygen requirement. Additional signs and symptoms include slowed growth, recurrent lower respiratory tract
•5 infections, lower extremity clubbing, symptoms of congestive heart failure, loud 5 2 , and bounding peripheral pu lses . Risk
•6 factors for PDA include high altitude, maternal first-trimester ru bella, and premature birth . The ductus arteriosus normally
•7 closes w ithin 10-15 hours of birth, but occasionally occurs around day 2 of life . Echocardiography commonly shows left atrial
and left ventricular enlargement. A color flow Doppler showing blood flow from the aorta into the pulmonary artery is
•8
diagnostic. If diagnosed in infancy, indomethacin (a prostaglandin synthesis inhibitor) is given to promote PDA closure. If
•9 indomethacin fails or the child is greater than 6-8 months in age, surgical closure (PDA ligation) is pursued . If the PDA is not
• 10 closed, the patient is at increased risk of pulmonary congestion and hypertension, and eventual heart failure .
Patent ductus arteriosus Indometacin Prostaglandin Echocardiography Pulmonary artery Congestive heart failure Aorta Pulse pressure Ductus arteriosus Heart failure Hypertension
• 11
Preterm birth Pregnancy Oxygen Heart Pulmonary edema Ventricle (heart)
• 12
. 13 B is not correct. 170/o chose this .
. 14 Prostaglandin E1 (PGE 1) prevents a patent ductus arteriosus (PDA) from closing . PGE1 is given to patients with transposition
• 15
of the great vessels who have a PDA in order to facilitate blood exchange between the pulmonary and systemic circulations .
Without a PDA or septal defect, transposition is not compatible with life. PGE 1 may also be given to ductus-dependent
• 16
patients w ith coarctation or interruption of the aortic arch, or pulmonic valve obstruction . In this patient with an isolated PDA,
• 17 PGE 1 should not be administered, as the PDA is not critical to life and is associated with multiple complications in later life .
• 18 Transposition of the great vessels Patent ductus arteriosus Pulmonary valve Prostaglandin Ductus arteriosus Aortic arch Great vessels

• 19 C is not correct. 20/o chose this.


• 20 Aortic valve replacement is pursued in patients with a critical obstruction, normally due to a congenital bicuspid aortic valve in
• 21 younger patients or a calcified aortic valve in the elderly. Congenital bicuspid aortic valve is three to four times more likely in
• 22
males than females . Patients normally present with signs and symptoms in adolescence or adulthood, when critical aortic
stenosis develops . Strenuous activity and competitive sports should be avoided . Signs include an early systolic ejection
• 23
murmur heard best at the right sternal border w ith rad iation into the carotid arteries, an audible S3 and/or S4, and pulsus
• 24

Lock
s
Suspend
0
End Block
Item: 1 of 24 ~ 1 • Mark -<] C> Jill ~· ~J
QID: 237 16 ..1 Previous Next Lab'V!I!ues Notes Calculator

1
C i s not correct. 2% chose this .
•2
Aortic valve replacement is pursued in patients with a crit ical obstruction, normally due to a congenita l bicuspid aortic valve in
•3 younger patients or a calcified aortic valve in the elderly. Congenital bicuspid aortic valve is three to four t imes more li kely in
•4 males than females . Patients normally present with signs and symptoms in adolescence or adulthood, when critical aortic
stenosis develops . Strenuous activity and competit ive sports should be avoided . Signs include an early systolic ej ection
•5
murmur heard best at the right sternal border w ith rad iation into the carotid arteries, an audible S3 and/or 5 4 , and pulsus
•6 parvus et tardus . Symptoms associated with severe aortic stenosis include dyspnea, angina, and syncope . In patients with
•7 peak systolic gradient >50 mm Hg or valve orifice < 1.0 cm 2 , aortic valve replacement should be considered . In asymptomatic
•8 children, adolescents, or young adults without calcific lesions, balloon valvuloplasty can be considered .
Dyspnea Bicuspid aortic valve Aortic valve Pulsus tardus et parvus Aortic valve replacement Systole Stenosis Syncope (medicine) Aortic stenosis Angina pectoris
•9
Common carotid artery Carotid artery Congenital disorder Balloon valvuloplasty Asymptomatic Valve replacement Calcification Artery Aortic valvuloplasty Sternum Mitral valve
• 10
Heart murmur Heart valve repair
• 11

• 12 D i s not correct. SO/o chose this .


• 13 Close monitoring of patients with congenital defects is only appropriate management for small atrial septal defects (ASDs) or
• 14
ventricular septal defects (VSDs) . Signs of ASD include filxed splitting of the second heart sound, and a systolic murmur at the
left sternal border. ASDs are well tolerated in childhood but pu lmonary hypertension can occur later in life. Surgery is only
• 15 recommended for symptomatic and la rge defects. VSDs are the most common acyanotic heart lesion (25% of all congenital
• 16 heart disease) and are associated w ith a harsh holosysto,lic murmur at the left sternal border. Most VSDs close in the first few
• 17 years of life . Most patients are asymptomatic, but patients with larger defects present w ith pulmonary congestion later in life .
Pulmonary hypertension Congenital heart defect Heart sounds Pulmonary edema Systole Hypertension Interventricular septum Sternum Asymptomatic Congenital disorder
• 18
Cardiovascular disease Split 52 Ventricle (heart) Lesion Atrium (heart) Heart murmur
• 19
• 20 E i s not correct. 2 0/o c hose th is .
• 21 Surgical correction with arterial or atrial switch (Senning or Mustard procedures) is undertaken in infants born w ith
t ransposition of the great vessels . Patients with transposition develop symptoms of cyanosis, poor peripheral perfusion, and
• 22
tachypnea with no heart murmur within hours to days of birth w ith closure of the ductus arteriosus. In transposition, the
• 23 aorta is connected to the right ventricle and the pu lmonary artery to the left ventricle, resulting in parallel and independent
• 24 ,.. "" '''-'""""''!!~> .." ~ ......... ,..\,,..p ..... .....,.;,.. ,..; .. ,.., .l~p ;,.., ... ,.. 1\. ~'"'-'"' +- ~ I ~ ..... ~............ ,... .. - "'!t.p ..... ..,p ,..,! , ,,..f. , ,,.. ~ ..p ........ ;,....,..,,,.. fOr\/\\ ;,.. ,.. ..;p;,.."'!t. l ~....... I iF ..... r .... 4='<!!t. .... f.,.. u1 i+-l.-.. 01""\J\t'"'

Lock
s
Suspend
0
End Block
Item: 1 of 24 ~ 1 • Mark -<] C> Jill ~· ~J
QID: 237 16 ..1 Previous Next Lab'V!I!ues Notes Calculator

1 heart disease) and are associated w ith a harsh holosysto,lic murmur at the left sternal border. Most VSDs close in the first few
•2 years of life . Most patients are asymptomatic, but patients with larger defects present w ith pulmonary congestion later in life .
Pulmonary hypertension Congenital heart defect Heart sounds Pulmonary edema Systole Hypertension Interventricular septum Sternum Asymptomatic Congenital disorder
•3
Cardiovascular disease Split 52 Ventricle (heart) Lesion Atrium (heart) Heart murmur
•4
•5 E is not correct. 20/o chose this .
•6 Surgical correction with arterial or atrial switch (Senn ing or Mustard procedures) is undertaken in infants born w ith
•7
transposition of the great vessels . Patients with transposition develop symptoms of cyanosis, poor peripheral perfusion, and
tachypnea with no heart murmur within hours to days of birth w ith closure of the ductus arteriosus. In transposition, the
•8 aorta is connected to the right ventricle and the pulmonary artery to the left ventricle, resulting in parallel and independent
•9 pulmonary and systemic circulations . A septal defect or patent ductus arteriosus (PDA) is critical for life . Infants with PDAs
• 10 are treated with prostaglandin E1 to keep the PDA open until surgery. Preferred treatment involves surgical arterial switch of
the aorta and pulmonary arteries in infancy. In older patit ents, intracardiac repair involving intra-atrial switches ("baffles") can
• 11
be done .
• 12 Transposition of the great vessels Cyanosis Patent ductus arteriosus Prostaglandin Tachypnea Heart murmur Pulmonary artery Atrial switch Aorta Ductus arteriosus Ventricle (heart)

• 13 Artery Jatene procedure Atrium (heart) Great vessels Perfusion


• 14

• 15
Bottom Line:
• 16
• 17
Patent ductus arteriosus presents with wide pulse pressure, continuous machine-like murmur, and normal oxygen
requirement. If it fails to close, indomethacin (a prostaglandin synthesis inhibitor) is given to promote closure and prevent
• 18
heart failure.
• 19 Patent ductus arteriosus lndometacin Prostaglandin Ductus arteriosus Pulse pressu re Heart failure Heart murmur Oxygen

• 20
• 21
• 22 References:
FA Step 2 CK 9th ed pp 373-375
• 23
FA Step 2 CK 8th ed pp 357-358
• 24

Lock
s
Suspend
0
End Block
Item: 2 of 24 ~ 1 • Mark -<] C> Jill ~· ~J
QID: 24043 ..1 Previous Next Lab'V!I!ues Notes Calculator

1 •
An 8-year-old boy with chron ic kidney disease is brought by his parents to the emergency departmen t . The child says he ~~AI
•2
has been feeling weak for the past several hours. A basic metabolic panel shows a sodium level of 142 mEq/L, potassium
•3 of 7 .2 mEq ./L, and chloride of 100 mEq/L. ECG demonstrates peaked T waves in leads II, III, V3, and V4 .
•4
•5 What is the best next step in management?
•6
•7 A. Hemodia lysis
•8
B. Intravenous calcium gluconate
•9
• 10
C. Intravenous dextrose alone

• 11 D. Intravenous dextrose and insulin


• 12
E. Nebu lized albuterol
• 13
• 14
• 15
• 16
• 17
• 18
• 19
• 20
• 21
• 22
• 23
• 24

Lock
s
Suspend
0
End Block
Item: 2 of 24 ~ 1 • Mark -<] C> Jill ~· ~J
QID: 24043 ..1 Previous Next Lab'V!I!ues Notes Calculator

2 The correct answer i s B. 6 30/o chose th is.


•3 The clinical scenario and the ECG point to card iac manifestations of hyperkalemia . Although the patient is experiencing
muscle weakness, arrhythmias are the primary cause of death with hyperkalemia and must be addressed immediately.
•4
Calcium gluconate is the best cardioprotective agent for hyperkalemia . High levels of potassium lead to depolarization of the
•5 resting membrane potential. This causes inactivation of sodium channels and decreased membrane excitability. The calcium in
•6 calcium gluconate antagonizes the membrane actions of hyperkalemia . This protective effect beg ins within m inutes of infusion
Hyperkalemia Calcium gluconate Membrane potential Depolarization Electrocardiogr aphy calcium Sodium Sodium channel Cardiac arrhythmia Muscle weakness Resting potential
•7
Potassium Muscle Cell membrane
•8
•9 A i s not correct. 150/o chose this .
• 10 Hemodialysis is one of the defin it ive ways of removing potassium from the body. However, because it takes significantly
• 11
longer to in it iate than intravenous calcium, hemodialysis is not the preferred next step in management.
Hemodialysis Intravenous therapy Calcium Potassium
• 12
• 13 C i s not correct. 30/o chose this .
• 14 Intravenous dextrose is not an adequate treatment for hyperkalemia .
Hyperkalemia Glucose Intravenous therapy
• 15
• 16 D i s not correct. 180/o chose this .
• 17 Insulin drives potassium into cells, causing a temporary decrease in serum potassium levels . However, insulin is only a
temporizing measure and does not offer the cardioprotective benefit of intravenous calcium . Dextrose must be given along
• 18
w ith insulin to prevent hypoglycemia .
• 19 Hypoglycemia Insulin Glucose Potassium Intravenous therapy Blood plasma Calcium Serum (blood)
• 20
E i s not corr ect. 1 0/o c hose th is .
• 21
Nebulized albuterol may be used to reduce potassium levels by driv ing potassium into cells. Like insuli n, it provides only a
• 22 temporary reduction in serum potassium .
• 23 Insulin Salbutamol Potassium Blood plasma Nebulizer

• 24

Lock
s
Suspend
0
End Block
Item: 2 of 24 ~ 1 • Mark -<] C> Jill ~· ~J
QID: 24043 ..1 Previous Next Lab'V!I!ues Notes Calculator
• •
1

2
Hemodialysis is one of the defin it ive ways of removing potassium from the body. However, because it takes significantly
longer to init iate than intravenous calcium, hemodialysis is not the pr eferred next step in management .
•3 Hemodialysis Intravenous therapy Calcium Potassium
•4
C i s not correct. JO/o chose this .
•5
Intravenous dextrose is not an adequate treatment for hyperkalemia .
•6
Hyperkalemia Glucose Intravenous therapy
•7
D i s not correct. 180/o chose this .
•8
Insulin drives potassium into cells, causing a temporary decrease in serum potassium levels . However, insulin is only a
•9
temporizing measure and does not offer the cardioprotective benefit of intravenous calcium . Dextrose must be given along
• 10 w ith insulin to prevent hypoglycemia .
• 11 Hypoglycemia Insulin Glucose Potassium Intravenous therapy Blood plasma Calcium Serum (blood)

• 12 E i s not corr ect. 1 0/o chose th is .


• 13 Nebulized albuter ol may be used to reduce potassium levels by driv ing potassium into cells. Like insuli n, it provides only a
• 14 temporary reduction in serum potassium .
Insulin Salbutamol Potassium Blood plasma Nebulizer
• 15
• 16
• 17 Bottom line :
• 18
Calcium gluconate is the best cardioprotective agent for hyperkalemia and stabilizes ca rdiac membranes.
• 19 Hyperkalemia Calcium gluconate Calcium Gluconic acid

• 20
• 21
• 22 References:
FA Step 2 CK 9th ed pp 470-471
• 23
FA Step 2 CK 8th ed pp 441-442
• 24

Lock
s
Suspend
0
End Block
Item: 3 of 24 ~ 1 • Mark -<] C> Jill ~· ~J
QID: 23644 ..1 Previous Next Lab'V!I!ues Notes Calculator

1 •
An 11-month-old boy w ith a chromosome 22q11 deletion is brought to the clinic by his mother. Over the past few weeks, ~ ~A I
2 the mother states her son seems to turn blue and have di fficu lty breath ing whenever he attempts to stand up. On
•3 presentation, the boy has an unusually long face, high and broad nasal bridge, low-set ma lformed ears, and a short
•4 philtrum . Physical examination is normal except for a murmur upon auscu ltation of the heart, and fol low-up cardiac imaging
•5
demonstrates an overriding aorta .
•6
Why would this patient have appeared clinically acyanotic before his presentation despite his heart defect?
•7
•8
•9
A. Atria l septal defect

• 10 B. Infants with tetra logy of Fa llot are always cyanotic


• 11
C. Large ventricu lar septa l defect, minimal right ventricular outflow obstruction
• 12
D. Large ventricu lar septa l defect, signif icant right ventricular outflow obstruction
• 13
• 14 E. Massive right ventricular hypertrophy
• 15 F. Overriding aorta
• 16
• 17
• 18
• 19
• 20
• 21
• 22
• 23
• 24

Lock
s
Suspend
0
End Block
Item: 3 of 24 ~ 1 • Mark -<] C> Jill ~· ~J
QID: 23644 ..1 Previous Next Lab'V!I!ues Notes Calculator

2 The correct answer is C. 640/o chose this.


3 In the case of a large ventricular septal defect (VSD), t he pressure in the left ventricle appr oximates that of the right
•4 ventricle (RV). The flow across the VSD is then determined by the path of least resistance. I f the RV outflow obstruction is
minimal, then blood will be shunted left to right, which will not cause cyanosis.
•5
Ventricular septal defect Cyanosis Ventride (heart) Ventricular system
•6
•7
A is not correct. 100/o chose this.
An atrial septal defect (ASD ) normally produces left-to-ri ght shunting due to lower right-sided pressures. However, given the
•8
typical pulmonary stenosis of tetralogy of Fallot, the ri ght-sided pressures are elevated so that a patient that also has an ASD
•9 would most likely still be cyanotic with a right-to-left shunt. ASD is not part of the pathology in tetralogy of Fallot.
• 10 Atrial septal defect Tetralogy of Fallot Right-to-left shunt Pulmonic stenosis Cyanotic heart defect Cyanosis Atrium (heart) Stenosis Shunt (medical) Pathology

• 11 B is not correct. 4 0/o chose this .


• 12 If the ventricular septal defect is sufficiently large and the right ventricle outflow obstruction is minimal, then a left-to- right
• 13 shunt w ill exist, which is not a cyanotic condition .
Ventricular septal defect Ventride (heart) Cyanotic heart defect Cyanosis Cardiac shunt Ventricular system Shunt (medical)
• 14

• 15 D is not correct. 140/o chose this .


• 16 In the case of a large ventricular septal defect (VSD), t he pressure in the left ventricle appr oximates that of the right
• 17 ventricle (RV). The flow across the VSD is then determined by the path of least resistance. As the RV outflow becomes
sufficiently obstructed, the outflow pressure becomes lar ger than that in the aorta and blood is shunted .
• 18
Ventricular septal defect Ventride (heart) Aorta Ventricular system
• 19
• 20
E is not correct. 4 0/o chose th is .
Right ventricle hypertrophy suggests a contraction against a massive right ventricle outflow obstruction, w hich itself will likely
• 21
cause a right-to-left sh unt and produce cyanosis .
• 22 Cyanosis Right-to-left shunt Ventride (heart) Hypertrophy Ventricular system Shunt (medical)
• 23
F is not correct. 4 0/o chose this.
• 24

Lock
s
Suspend
0
End Block
Item: 3 of 24 ~ 1 • Mark -<] C> Jill ~· ~J
QID: 23644 ..1 Previous Next Lab'V!I!ues Notes Calculator

1 • shunt w ill exist, which is not a cyanotic condition .


Ventricular septal defect Ventricle (heart) Cyanotic heart defect Cyanosis Cardiac shunt Ventricular system Shunt (medical)
2
3 D is not correct. 140/o chose this.
•4 In the case of a large ventricular septal defect (VSD), t he pressure in the left ventricle approximates that of the right
•5 ventricle (RV) . The flow across the VSD is then determined by the path of least resistance. As the RV outflow becomes
sufficiently obstructed, the outflow pressure becomes lar ger than that in the aorta and blood is shunted .
•6
Ventricular septal defect Ventricle (heart) Aorta Ventricular system
•7
E is not correct. 40/o chose this .
•8
Right ventricle hypertrophy suggests a contraction against a massive right ventricle outflow obstruction, w hich itself will likely
•9
cause a right-to-left shunt and produce cyanosis .
• 10 Cyanosis Right-to-left shunt Ventricle (heart) Hypertrophy Ventricular system Shunt (medical)

• 11
F is not correct. 40/o chose this .
• 12
An overriding aorta is part of the diagnosis of tetralogy of Fallot but does not contribute to w hether the shunt across the
• 13 ventricular septal defect is left to ri ght or right to left.
• 14 Tetralogy of Fallot Ventricular septal defect Overriding aorta Aorta Shunt (medical)

• 15
• 16
Bottom line:
• 17
An infant with tetralogy of Fallot w ill be acyanotic if she or he has a large ventricular septal defect and minimal right
• 18
ventricular outflow obstruction .
• 19 Tetralogy of Fallot Ventricular septal defect Acyanotic heart defect Tetralogy

• 20
• 21
• 22 References:
FA Step 2 CK 9th ed pp 377; 378
• 23
FA Step 2 CK 8th ed pp 359-360
• 24

Lock
s
Suspend
0
End Block
Item: 4 of 24 ~ 1 • Mark -<] C> Jill ~· ~J
QID: 237 19 ..1 Previous Next Lab'V!I!ues Notes Calculator

1 •
A 6-year-old boy is evaluated as a new patient in the peduatrics clinic. The mother reports a normal pregnancy and den ies
2 any complications or subsequent hospitalizations . All immunizations are up to date . On physical examination, however, a
3 III/VI harsh systolic ejection murmur is heard at the right upper sternal bor der, preceded by a cl ick at the left lower
•4 sternal border. The physician suspects aortic stenosis, and obtain an echocardiogram; subsequently she tells the boy's mother
•5
that her child w ill likely need balloon valvuloplasty or surgery at some point. The physician also advises her that her child should
not participate in competit ive sports .
•6
•7
Of the following late symptoms associated with this child's condition, w hich one carries the poorest prognosis?
•8
•9
A. Ang ina
• 10
• 11
B. Dyspnea
• 12 C. Fever
• 13
D. Hypertension
• 14
• 15 E. Syncope
• 16
• 17
• 18
• 19
• 20
• 21
• 22
• 23
• 24

Lock
s
Suspend
0
End Block
Item: 4 of 24 ~ 1 • Mark -<] C> Jill ~· ~J
QID: 237 19 ..1 Previous Next Lab'V!I!ues Notes Calculator

2
The correct answer i s B. 3 5 0/o chose th is.
3
Congenital bicuspid aortic valve occurs in 2% of the general population , and is the most common cause of aortic stenosis in
4 individuals <70 years old. Childr en with a bicuspid aortic valve develop calcifications earlier in life, sometimes beginning in
•5 the second decade of life. While children ar e often asymptomatic, almost all patients wil l develop symptoms in adulthood
•6
necessitating va lve replacement, with the peak incidence between ages 40 and 60 years . In severe or crit ical aort ic stenosis,
infants < 1 year old can develop symptoms of hea rt failure. The development of heart fa ilure (with symptoms such as
•7
dyspnea) is most severe. Of the 50% of patients who present with dyspnea, half will die within 2 years without surg ical valve
•8 replacement. Patients may either receive a mechanical aortic valve, or undergo autograft valve replacement, with
•9 transplantation of the native pulmonary valve to the aortic position ( Ross procedure) . Children with aortic stenosis should not
participate in competitive sports due to increased obstruction of the outflow t ract.
• 10
Ross procedure Dyspnea Aortic stenosis Pulmonary valve Bicuspid aortic valve Aortic valve Autotransplantation Stenosis Congenital disorder Asymptomatic Mitral valve Heart failure
• 11
Valve replacement
• 12
• 13
A i s not correct. 14% chose this .
• 14
In the 35% of patients with aortic stenosis who present with angina, half will die with in 5 year s without sur gical va lve
replacement .
• 15 Aortic stenosis Stenosis Angina pectoris Valve replacement
• 16
C i s not correct. 5 % chose this .
• 17
Patients with congenital bicuspid aort ic valve are at increased risk for infective endocarditis, which can present with fever.
• 18
Infective endocardit is is normally successfully t reated with intravenous antibiotics and is not an indication for valve
• 19 replacement .
• 20 Infective endocarditis Bicuspid aortic valve Aortic valve Endocarditis Congenital disorder Mitral valve Valve replacement Intravenous therapy Antibiotics Fever

• 21 D is not correct. 8 % chose this .


• 22 Patients with congenital bicuspid aort ic valve often develop hypertension due to left ventricular obstruction . Systolic
• 23 hypertension develops first, but as the severity of stenosis increases, the left ventricle hypertrophies and becomes less
• 24
compliant, result ing in diastolic hypertension as well as systolic hypertension . While hypertension is common, it is not

Lock
s
Suspend
0
End Block
Item: 4 of 24 ~ 1 • Mark -<] C> Jill ~· ~J
QID: 237 19 ..1 Previous Next Lab'V!I!ues Notes Calculator
I I - • f I •
1
Patients with congen ital bicuspid aortic valve are at increased risk for infective endocard it is, w hich can present with fever.
2
Infective endocarditis is normally successfully treated w ith intravenous antibiotics and is not an indication for valve
3 replacement.
4 Infective endocarditis Bicuspid aortic valve Aortic valve Endocarditis Congenital disorder Mitral valve Valve replacement Intravenous therapy Antibiotics Fever

•5
D is not correct. SOfo chose this.
•6 Patients with congen ital bicuspid aortic valve often develop hypertension due to left ventricular obstruction . Systolic
•7 hypertension develops first, but as the severity of stenosis increases, the left ventricle hypertroph ies and becomes less
•8 compliant, resulting in diastolic hypertension as well as systolic hypertension . While hypertension is common, it is not
associated w ith poorer prognosis .
•9
Bicuspid aortic valve Aortic valve Hypertension Ventricle {heart) Systole Diastole Stenosis Congenital disorder Mitral valve Heart Systolic hypertension Ventricular system Prognosis
• 10
E is not correct. 38% chose this.
• 11
Of the 15% of patients with aortic stenosis who present with syncope, ha lf will die within 3 years without aortic valve
• 12
replacement .
• 13 Aortic stenosis Aortic valve Syncope (medicine) Stenosis Aortic valve replacement Valve replacement
• 14

• 15
Bottom Line:
• 16
• 17
Congenital bicuspid aortic valve occurs in 2% of the genera l popu lation, and is the most common cause of aortic stenosis in
individuals <70 years old. Of the SO% of patients who present with dyspnea, ha lf will die with in 2 years without surgical
• 18
valve replacement.
• 19 Dyspnea Bicuspid aortic valve Aortic stenosis Aortic valve Congenital disorder Stenosis Valve replacement Mitral valve

• 20
• 21
• 22 References:
FA Step 2 CK 9th ed pp 43; 20; 21
• 23
FA Step 2 CK 8th ed p 42
• 24

Lock
s
Suspend
0
End Block
Item: 5 of 24 ~ 1 • Mark -<] C> Jill ~· ~J
QID: 24291 ..1 Previous Next Lab'V!I!ues Notes Calculator

1 •
Shortly after birth, a newborn is found to be tachypneic, hypoxemic, and cyanotic. Cardiac auscultation reveals a ~ ~A I
2 continuous machine-like murmur at the sternal border in the second left intercostal space. A single S2 is present. X-ray of
3 the chest (image provided) revea ls a narrow heart base, an egg-shaped cardiac sil houette, and increased pulmonary
4 vasculature markings. The patient is also noted to have a patent ductus arteriosus in addition to the more severe cardiac
•5
anomaly present .
•6
•7
•8
•9
• 10
• 11

• 12
• 13
• 14
• 15
• 16
• 17
• 18
• 19
• 20
• 21
• 22
Image courtesy of Wikimedia
• 23
• 24

Lock
s
Suspend
0
End Block
2
3
4
•5
.6
.7
·8
•9
• 10
• 11
• 12
Image courtesy of Wikimedia
• 13
• 14
Which of the following conditions does this patient have?
• 15
• 16
A. Tetralogy of Fallot
• 17
• 18 B . Total anoma lous pulmonary venous return
• 19
C. Transposition of the great vessels
• 20
D . Tricuspid atresia
• 21
• 22 E. Truncus arteriosus
• 23

• 24

a
Lock
s
Suspend
8
End Block
Item: 5 of 24 ~ 1 • Mark -<] C> Jill ~· ~J
QID: 24291 ..1 Previous Next Lab'V!I!ues Notes Calculator

1
The correct answer is C. 480/o chose this.
2
This infant has transposition of the great vessels (TGV), which is the most common cyanotic congenital heart defect in
3 newborns. In TGV, the aorta is connected to the right ventricle and the pulmonary artery is connected to the left ventricle.
4 This creates separate pulmonary and system ic circulations, wh ich is incompatible w ith life unless the patient has a secondary
5 card iac defect (like the patent ductus arteriosus [PDA] this patient has) or a septal defect. The machine-like murmur and a
loud 5 2 are characteristic of PDA. Cyanosis and critical il lness are most often present immed iately after birth . I n addition to
•6
tachypnea, cyanosis, and hypoxemia, some patients have signs of congestive heart failure (CH F). The radiographic findings
•7 described (a narrow heart base and absence of the main pulmonary artery segment, resulting in an egg-shaped silhouette,
•8 and increased vascular markings) are present at birth . Patients should be treated with intravenous prostaglandin E1 to
•9 maintain the PDA. If the PDA cannot be mainta ined, or if a patient has a small atrial septal defect (A5D) before the heart can
be surgically repaired, balloon atrial septostomy shou ld be performed to enlarge or create an A5D. Definitive management is
• 10
surgical repair via an atrial or arterial switch procedure.
• 11 Atrial septal defect Transposition of the great vessels Cyanosis Patent ductus arteriosus Congenital heart defect Tachypnea Prostaglandin Hypoxemia Aorta Pulmonary artery

• 12 Ventricle (heart) Heart failure Congestive heart failure Cyanotic heart defect Ouctus arteriosus Congenital disorder Great vessels Intravenous therapy Jatene procedure Heart murmur

• 13 Atrium (heart) Radiography


• 14
A is not correct. 130/o chose this.
• 15
Tetralogy of Fallot is another of the cyanotic congen ital h,e art defects . Tetralogy of Fallot is associated with pulmonary
• 16 stenosis, overriding aorta, right ventricular hypertrophy, and ventricular septal defect. These children often present later, in
• 17 infancy or early childhood rather than immediately after birth . Infants may present with congestive heart fail ure, as
• 18 manifested by diaphoresis and fatigue w ith feeding or tachypnea. Cyanosis usually develops within the first 2 years of life,
and the severity of the cyanosis is related to the severity of the pulmonary stenosis. Children may experience "tet spells" in
• 19
which they become hypoxemic and cyanotic, lead ing them to squat for relief, which increases systemic vascula r resistance
• 20 and thus perfusion . Cardiac examination revea ls a systolic ejection murmur present at the left upper sternal border, a single
• 21 5 2 , and a right ventricular heave . X-ray of the chest reveals a boot-shaped cardiac silhouette w ith decreased pulmonary
• 22
vascular markings. Definitive treatment involves surgica l repair, although some children undergo palliative balloon atrial
septostomy procedures to decrease the severity of their symptoms before surgica l repair is possible .
• 23 Tetralogy of Fallot Ventricular septal defect Cyanosis Overriding aorta Perspiration Tachypnea Heart failure Pulmonic stenosis Right ventricular hypertrophy Chest radiograph Aorta
• 24

Lock
s
Suspend
0
End Block
Item: 5 of 24 ~ 1 • Mark -<] C> Jill ~· ~J
QID: 24291 ..1 Previous Next Lab'V!I!ues Notes Calculator

1
A is not correct. 130/o chose this.
2
Tetralogy of Fallot is another of the cyanotic congen ital h,e art defects . Tetralogy of Fallot is associated with pulmonary
3
stenosis, overriding aorta, right ventricular hypertrophy, and ventricular septal defect. These children often present later, in
4 infancy or early childhood rather than immediately after birth . I nfants may present with congestive heart fail ure, as
5 manifested by diaphoresis and fatigue w ith feeding or tachypnea . Cyanosis usually develops within the first 2 years of life,
•6 and the severity of the cyanosis is related to the severity of the pulmonary stenosis. Children may experience "tet spells" in
which they become hypoxemic and cyanotic, lead ing them to squat for relief, wh ich increases systemic vascular resistance
•7
and thus perfusion . Cardiac examination revea ls a systolic ejection murmur present at the left upper sternal border, a single
•8 5 2 , and a right ventricular heave . X-ray of the chest revea ls a boot-shaped cardiac silhouette w ith decreased pulmonary
•9 vascular markings. Definitive treatment involves surgica l repair, although some children undergo palliative balloon atrial
• 10
septostomy procedures to decrease the severity of their symptoms before surgica l repair is possible .
Tetralogy of Fallot Ventricular septal defect Cyanosis Overriding aorta Perspiration Tachypnea Heart failure Pulmonic stenosis Right ventricular hypertrophy Chest radiograph Aorta
• 11
Congestive heart failure Hypoxia (medical) Hypoxemia Cyanotic heart defect Systole Stenosis Ventricular hypertrophy Congenital disorder Congenital heart defect Hypertrophy
• 12
Vascular resistance Blood vessel Cardiac examination X-ray Fatigue (medical) Parasternal heave Atrium (heart) Perfusion Palliative care Heart murmur
• 13
• 14 B is not correct. 150/o chose th is .
• 15 In total anoma lous pulmonary venous return, both the pulmonary veins and the systemic veins return to the right atrium .
The pulmonary veins can either drain directly into the right atrium, or can anastomose with various systemic veins or the
• 16
coronary sinus and enter the right atrium through one of those vessels. An atrial septal defect, patent foramen ovale, or
• 17 patent ductus arteriosus is necessary for survival. Patients present with cyanosis and often progress to congestive heart
• 18 failure within the first year of life. The severity of the presentation relates to whether or not pulmonary hypertension is
• 19 present. Cardiac auscultation reveals a systolic ejection murmur at the right or left sterna l border, as well as a mid-diastolic
rumble in the tricuspid va lve reg ion . X- ray of the chest reveals increased pulmonary markings with a prominent pu lmonary
• 20
artery arch, as well as right ventricular hypertrophy. I n patients w ith pulmonary hypertension, pulmonary edema can also be
• 21 present. Surgical correction should be immediate if the patient has severe pulmonary hypertension, but can be delayed for
• 22 several years if the patient does not exh ibit signs of pulmonary hypertension .
Atrial septal defect Patent foramen ovale Cyanosis Patent ductus arteriosus Heart failure Atrium (heart) Pulmonary edema Chest radiograph Pulmonary artery Congestive heart failure
• 23
Coronary sinus Right ventricular hypertrophy Tricuspid valve Auscultation Ductus arteriosus Foramen ovale (heart) Ventricular hypertrophy Heart sounds Pulmonary hypertension
• 24

Lock
s
Suspend
0
End Block
Item: 5 of 24 ~ 1 • Mark -<] C> Jill ~· ~J
QID: 24291 ..1 Previous Next Lab'V!I!ues Notes Calculator

1
B is not correct. 150/o chose th is.
2
In total anoma lous pulmonary venous return, both the pulmonary veins and the systemic veins return to the right atrium .
3
The pulmonary veins can either drain directly into the ri ght atrium, or can anastomose with various systemic veins or the
4 coronary sinus and enter the right atrium through one of those vessels. An atrial septal defect, patent foramen ovale, or
5 patent ductus arteriosus is necessary for survival. Patients present with cyanosis and often progress to congestive heart
•6 failure within the first year of life. The severity of the presentation relates to whether or not pulmonary hypertension is
present. Cardiac auscultation reveals a systolic ej ection murmur at the right or left sterna l border, as well as a mid-diastolic
•7
rumble in the t ricuspid va lve region . X- ray of the chest reveals increased pulmonary ma rkings with a prominent pu lmonary
•8 artery arch, as well as right ventricular hypertrophy. I n patients w ith pulmonary hypertension, pulmonary edema can also be
•9 present. Surgical correction should be immediate if the patient has severe pulmonary hypertension, but can be delayed for
• 10
several years if the patient does not exh ibit signs of pulmonary hypertension .
Atrial septal defect Patent foramen ovale Cyanosis Patent ductus arteriosus Heart failure Atrium (heart) Pulmonary edema Chest radiograph Pulmonary artery Congestive heart failure
• 11
Coronary sinus Right ventricular hypertrophy Tricuspid valve Auscultation Ductus arteriosus Foramen ovale (heart) Ventricular hypertrophy Heart sounds Pulmonary hypertension
• 12
Systole Anastomosis Hypertension Hypertrophy Pulmonary vein Anomalous pulmonary venous connection Edema Vein Sternum X-ray Tricuspid atresia Heart murmur
• 13
• 14 D is not correct. 100/o chose this .
• 15 Tricuspid atresia is a cyanotic congen ital heart lesion . In tricuspid atresia there is no communication between the right atrium
and the right ventricle . Most patients have an associated atrial septal defect (ASD), and patients have a complete right-to-left
• 16
atrial shunt. On cardiac auscu ltation, the murmu r associated w ith an ASD (a systolic ej ection murmur best heard at the upper
• 17 left sternal border) can be appreciated . Most patients also have a loud S1 . X-ray of the chest reveals cardiomegaly, with a
• 18 prominent and enlarged right atrium as well as decreased pulmonary vascular markings. Definitive treatment is via a three-
• 19 stage surgical repair, w ith the first stage, a Glenn (a cavopulmonary anastomosis) or Blalock-Taussig (subclavian-to-
pulmonary artery) shunt, being performed with in the first year of life .
• 20 Atrial septal defect Tricuspid atresia Cardiomegaly Auscultation Chest radiograph Cyanotic heart defect Atrium (heart) Cyanosis Ventricle (heart) Heart sounds Systole
• 21
Congenital disorder Atresia Tricuspid valve Lesion Anastomosis Sternum X-ray Shunt (medical) Blood vessel Heart murmur
• 22
E is not correct. 14% chose this .
• 23
Truncus arteriosus is a cyanotic congen ital heart defect that is much less common than tetralogy of Fallot or transposition of
• 24

Lock
s
Suspend
0
End Block
Item: 5 of 24 ~ 1 • Mark -<] C> Jill ~· ~J
QID: 24291 ..1 Previous Next Lab'V!I!ues Notes Calculator

1 prominent and enlarged right atrium as well as decreased pu lmonary vascular markings. Defin it ive treatment is via a three-
2 stage surgical repair, w ith the first stage, a Glenn (a cavopulmonary anastomosis) or Blalock-Taussig (subclavian-to-
pulmonary artery) shunt, being performed with in the first year of life.
3 Atrial septal defect Tricuspid atresia Cardiomegaly Auscultation Chest radiograph Cyanotic heart defect Atrium (heart) Cyanosis Ventricle (heart) Heart sounds Systole
4
Congenital disorder Atresia Tricuspid valve lesion Anastomosis Stemum X-ray Shunt (medical) Blood vessel Heart murmur
5
•6
E is not correct. 14% chose this.
•7
Truncus arteriosus is a cyanotic congenita l heart defect that is much less common than tetralogy of Fallot or transposit ion of
the great vessels. It consists of a single arteria l trunk arising from a single sem il unar valve. There are mu ltiple subtypes of
•8 truncus arteriosus that describe the branch ing pattern of the pu lmonary arteries from the main trunk. Patients present with
•9 poor feeding, tachypnea, and cyanosis . Cardiac auscu ltation revea ls an early systolic click and a holosystolic murmur heard
• 10 best at the third and fourth intercostal space, somewhat sim ilar to the murmur associated with a ventricular septal defect. A
thrill may also be present. X-ray of the chest revea ls engorged pulmonary arteries, and in half of cases, a right-sided aortic
• 11
arch . Truncus arteriosus is treated with surgical repair during the first several weeks of life .
• 12 Transposition of the great vessels Tetralogy of Fallot Ventricular septal defect Cyanosis Congenital heart defect Tachypnea Auscultation Aortic arch Pulmonary artery Chest radiograph

• 13 Truncus arteriosus Intercostal space Cyanotic heart defect Heart sounds Heart valve Systole Congenital disorder Great vessels Right-sided aortic arch Artery X-ray Heart murmur
• 14

• 15
Bottom Line:
• 16
• 17
In transposition of the great vessels, the aorta is connected to the right ventricle and the pu lmonary artery is connected to
the left ventricle. Th is defect is compatible with life on ly in the presence of a secondary card iac defect, such as a patent
• 18
ductus arteriosus .
• 19 Transposition of the great vessels Patent ductus arteriosus Pulmonary artery Aorta Ventricle (heart) Ductus arteriosus Great vessels

• 20
• 21
• 22 References:
FA Step 2 CK 9th ed pp 376-377
• 23
FA Step 2 CK 8th ed p 359
• 24

Lock
s
Suspend
0
End Block
Item: 6 of 24 ~ 1 • Mark -<] C> Jill ~· ~J
QID: 23649 ..1 Previous Next Lab'V!I!ues Notes Calculator

1 •
A 10-year-old boy with a history of an unknown heart condit ion is seeing a new dentist for the first time . He reports he ~~AI
2 had heart surgery at a young age, but he is unsure of any of the details. He currently den ies any fever or chills.
3
4 Which of the following requires endocarditis prophylaxis before any invasive dental work begins"
5
•6
A. History of Kawasaki disease
•7
B. Ostium secundum atria l septal defect
•8
•9 C. Physiologic murmur
• 10 D. Surgica lly repaired atrial septa l defect > 6 months ago
• 11
E. Tetralogy of Fallot with residual defect at prosthetk site
• 12
• 13
• 14
• 15
• 16
• 17
• 18
• 19
• 20
• 21
• 22
• 23
• 24

Lock
s
Suspend
0
End Block
Item: 6 of 24 ~ 1 • Mark -<] C> Jill ~· ~J
QID: 23649 ..1 Previous Next Lab'V!I!ues Notes Calculator

1 The correct answer i s E. 7 3% chose this .


2 Conditions that are considered high risk for bacterial endocardit is include a history of endocarditis, the presence of a
3 prosthetic valve, a history of unrepaired or partially repa ired cyanotic congenital heart disease ( eg, tetralogy of Fallot),
completely repa ired congenital heart disease with prosthetic material or device during the first six months after the
4
procedure, repaired congenital heart disease with residual defects, or cardiac transplantation recipients with cardiac valvular
5 disease . These conditions require antibiotic prophylaxis prior to dental procedures, which can cause transient bacteremia and
6 may lead to the seeding of an abnormal heart valve and subsequent endocarditis . The risk of infectious endocardit is is
•7 generally considered to be the highest for dental procedures that involve manipu lation of gingival tissue or perforation of the
oral mucosa, such as tooth extractions, drainage of a dental abscess, or even routine dental cleaning . Common valvu lar
•8
lesions for which antimicrobial prophylaxis is no longer recommended include bicuspid aortic valve, acquired aortic or m itral
•9 valve disease (including m it ral valve prolapse with regurgitation and prior valve repair), and hypertrophic cardiomyopathy
• 10 w ith latent or resting obstruction .
Tetralogy of Fallot Hypertrophic cardiomyopathy Mitral valve prolapse Congenital heart defect Bicuspid aortic valve Mitral insufficiency Aortic valve Endocarditis Antibiotics Bacteremia
• 11
Mitral valve Heart valve Cyanosis Artificial heart valve Cardiomyopathy Infective endocarditis Hypertrophy Heart transplantation Congenital disorder Regurgitation (circulation)
• 12
Cardiovascular disease Dental abscess Abscess Valvular heart disease ~1ucous membrane Ventricular hypertrophy Aortic valve replacement Preventive healthcare Prosthesis
• 13
• 14 A i s not correct. 3% chose this .
• 15 A history of Kawasaki disease is considered low risk by the American Heart Association and does not require antibiotic
• 16 prophylaxis. Kawasaki disease is associated with coronary artery aneurysms, wh ich can lead to myocardial ischemia,
infarction, and sudden death .
• 17 Kawasaki disease American Heart Association Antibiotics Ischemia Coronary artery disease Aneurysm Coronary circulation Preventive healthcare Infarction
• 18
B is not correct. 8 0/o chose this .
• 19
An ostium secundum atrial septal defect occurs when the septum secundum is not large enough to cover the ostium
• 20
secundum . This does not predispose to infectious endocard it is, and the American Heart Association does not recommend
• 21 antibacteria l prophylaxis.
• 22 Atrial septal defect Septum secundum Foramen secundum Infective endocarditis Atrium (heart) Endocarditis Septum American Heart Association Primary interatrial foramen

• 23 Preventive healthcare

• 24

Lock
s
Suspend
0
End Block
Item: 6 of 24 ~ 1 • Mark -<] C> Jill ~· ~J
QID: 23649 ..1 Previous Next Lab'V!I!ues Notes Calculator

1
B is not correct. 80/o chose this.
2
An ostium secundum atrial septal defect occurs when the septum secundum is not large enough to cover the ostium
3
secundum . This does not predispose to infectious endocarditis, and the American Heart Association does not recommend
4 antibacterial prophylaxis.
5 Atrial septal defect Septum secundum Foramen secundum Infective endocarditis Atrium (heart) Endocarditis Septum American Heart Association Primary interatrial foramen

6 Preventive healthcare

•7
C is not correct. 4% chose this.
•8
Physiologic murmurs can often be identified in the pediatric population, and as long as they are not suspicious for underlying
•9 pathology, they do not requ ire antibacterial prophylaxis . In an adult, such a murmur wou ld require echocardiography to rule
• 10 out structural abnormalities because physiologic murmurs are less common .
Echocardiography Pathology Antibiotics Preventive healthcare Pediatrics Physiology Heart murmur
• 11

• 12 D is not correct. 120/o chose this .


• 13 According to American Heart Association guidelines, surgically repai red ventral septal defects, atrial septal defects, and patent
• 14 ductus arteriosus do not requ ire prophylaxis unless the defect was surgically corrected recently ( eg, w ithin the past 6
months) .
• 15 Patent ductus arteriosus American Heart Association Ductus arteriosus Anatomical terms of location Preventive healthcare United States Atrium {heart)
• 16
• 17
• 18
Bottom line:
• 19 Conditions requiring antibiotic prophylaxis prior to dental procedures include a history of endocarditis, the presence of a
prosthetic valve, and a history of unrepaired cyanotic congenital heart disease .
• 20
Antibiotics Endocarditis Congenital heart defect Cyanosis Cyanotic heart defect Congenital disorder Cardiovascular disease Preventive healthcare Artificial heart valve Prosthesis
• 21 Antibiotic prophylaxis
• 22
• 23
• 24 y References:

Lock
s
Suspend
0
End Block
Item: 6 of 24 ~ 1 • Mark -<] C> Jill ~· ~J
QID: 23649

1
..1
- . . . .. Previous
. . Next
.... . . .
Lab'V!I!ues Notes
..
Calculator
. . .. . -. ... . . . .
secundum . This does not predispose to infectious endocarditis, and the American Hea rt Association does not recommend
2 a ntibacteria I prophylaxis.
3 Atrial septal defect Septum secundum Foramen secundum Infective endocarditis Atrium (heart) Endocarditis Septum American Heart Association Primary interatrial foramen

4 Preventive healthcare

5 C is not correct. 40/o chose this.


6 Physiologic murmurs can often be identified in the pediatric population , and as long as they are not suspicious for underlying
•7 pathology, they do not require antibacterial prophylaxis . In an adult, such a murmur would require echocar diography to rule
•8 out structural abnormalities because physiologic murmurs are less common .
Echocardiography Pathology Antibiotics Preventive healthcare Pediatrics Physiology Heart murmur
•9
• 10 D is not correct. 120/o chose this .
• 11 According to American Heart Association guidelines, sur gically r epaired ventral septal defects, atrial septal defects, and patent
ductus arteriosus do not require prophylaxis unless the defect was surgically corrected recently ( eg, w ithin the past 6
• 12
months) •
• 13 Patent ductus arteriosus American Heart Association Ductus arteriosus Anatomical terms of location Preventive healthcare United States Atrium (heart)
• 14
• 15
Bottom Line:
• 16
• 17
Conditions requiring antibiotic prophylaxis prior to dental procedures include a history of endocarditis, the pr esence of a
prosthetic valve, and a history of unrepaired cyanotic congenital heart disease .
• 18 Antibiotics Endocarditis Congenital heart defect Cyanosis Cyanotic heart defect Congenital disorder Cardiovascular disease Preventive healthcare Artificial heart valve Prosthesis
• 19 Antibiotic prophylaxis

• 20
• 21
• 22 References:
FA Step 2 CK 9th ed pp 372-373; 377; 378
• 23
FA Step 2 CK 8th ed p 356
• 24 y

Lock
s
Suspend
0
End Block
Item: 7 of 24 ~ 1 • Mark -<] C> Jill ~- ~J
QID: 23646 ..1 Previous Next Lab'V!I!ues Notes Calculator

1 •
A 12-year-old boy is brought to his pediatrician by his mother after complaining of episodes of feeling his heart racing . She ~~AI
2 reports that he has had these complaints for years, but that he often has somatic complaints and uses symptoms to
3 escape r esponsibility. On card iac auscultation, the boy has a grade III/VI holosystolic murmur heard at the right lower
4 sternal border. ECG reveals a supraventricula r tachycar dia . A repeat ECG after the tachycardia resolved is show n. An
echocar diogram is then performed, and the patient is diagnosed with Ebstein anomaly. His mother had a norma l pregnancy,
5
except that she was being t r eated for her bipolar disease at that t ime .
6
•7
•8
--0-- ._tl\ R~ -~r~lt---A"'r--N~ /u ~j
-~~~}~-A-jr~jA_J~J~~.~
•9
• 10 'r
• 11

• 12
- l -~'--rV>
ll
~ l·y 1\.
! I VJ
-..J\-1\
~ "
1 '-N,' -..~-
WJt· 'kJW
I ''--'! '9
~--v--
i
fv
1
• 13 ,

1
• 14
• 15
• 16
•-vtA ~t/I.Mtj\ _j0- ··~'-Vt.II~I~P'--N....., '---"'~--' '--"V "---"'l-11

I mage courtesy of James Heilman, MD


• 17
• 18
Which of the following is the pr eferr ed method of long-term management of this patient's arrhythmia?
• 19
• 20
A. Administer adenosine
• 21
• 22 B. Administer amiodarone
• 23
C. Adm inister metoprolol
• 24

Lock
s
Suspend
0
End Block
,..,
~ J~ I1I --v-
2
v
3
- l J.

~
I
4
.~
r' I ~,._A ~~ r I v-

5
I• I t
6
.7
I lf t

·8
•9 ~y-l I
r r-
I
• 10 Image courtesy of James Heilman, MD
• 11
• 12 Which of the follow ing is the preferred method of long-term management of this patient's arrhythmia?
• 13
• 14 A. Administer adenosine
• 15
B. Administer amiodarone
• 16
C. Administer metoprolol
• 17
• 18 D . Administer verapamil
• 19
E. Attempt a vagal maneuver
• 20
F. Attempt electrical cardioversion
• 21
• 22 G. Radiofrequency ablation
• 23

• 24

a
Lock
s
Suspend
8
End Block
Item: 7 of 24 ~ 1 • Mark -<] C> Jill ~· ~J
QID: 23646 ..1 Previous Next Lab'V!I!ues Notes Calculator

2 The correct answer is G. 3 9 0/o chose t hi s.


3 A risk factor for developing Ebstein anomaly (a spectrum of disorders of the tricuspid valve and right ventricle) is maternal
4
ingestion of lithium, used to treat bipolar disorder, during the first trimester. Generally, the valve leaflets are malformed,
divid ing the right ventricle into the proximal, or "atrialized," ventricle and distal, or "proper," ventricle. The presentation
5 varies considerably from heart failure from birth to palpitations much later in life, reflecting the wide variety of possible
6 anatomical variations . Earlier age at presentation is frequently associated with other cardiac lesions, such as atrial septal
7 defect and pulmonary stenosis. In adolescents and adults, arrhythmia is the most common presentation . Ebstein anomaly is
associated w ith atrial tachyarrhythm ia, some of which may be due to Wolff- Parkinson-White (WPW) syndrome since up to
•8
20% of patients have one or more accessory pathways that lie around the orifice of the malformed tricuspid va lve . The ECG
•9 shows delta wave and short PR interval characteristic of WPW. Aggressive efforts should be made to restore norma l sinus
• 10 rhythm in an attempt to prevent LV systolic dysfunction . Pharmacologic treatment of arrhythmias in any patient with WPW
• 11
should avoid AV noda l slowing agents, as these may make the arrhythmia worse. Procainamide would be the med ication of
choice. Definitive therapy of atrial tachyarrhythm ia includes catheter ablation . In patients with symptomatic WPW, catheter
• 12
ablation of t he accessory pathway is recommended .
• 13 Atrial septal defect Procainamide Bipolar disorder Cardiac arrhythmia Catheter ablation Cardioversion Pulmonic stenosis Tachycardia Tricuspid valve Sinus rhythm Ventricle (heart)
• 14 Electrocardiography Palpitations Systole Wolff-Parkinson-White syndrome catheter Heart failure Stenosis Risk factor Uthium PR interval Pharmacology Anatomical terms of location
• 15 Ablation Atrium (heart) Pharmaceutical drug
• 16
A i s not correct. 100/o chose this .
• 17
Adenosine blocks conduction at the atrioventricular (AV) node and may be used as short-term therapy for supraventricu lar
• 18 tachycardia (SVT) includ ing Wolff-Parkinson-White syndrome refractory to vaga l maneuvers. Vagal maneuvers shou ld be
• 19 performed first in acute treatment of SVT. Adenosine, while effective in acute management, has an extremely short half-life
• 20 (<lOs) . This patient has a known anatom ic abnorma lity classically associated with SVT and requ ires a more permanent
solution .
• 21
Wolff-Parkinson-White syndrome Supraventricular tachycardia Tachycardia Adenosine Vagal maneuver Atrioventricular node Vagus nerve
• 22
B is not correct. 120/o chose th is .
• 23
Amiodarone is used as short-term therapy in adults w ith a life-threatening wide-complex tachycardia such as ventricular
• 24

Lock
s
Suspend
0
End Block
Item: 7 of 24 ~ 1 • Mark -<] C> Jill ~· ~J
QID: 23646 ..1 Previous Next Lab'V!I!ues Notes Calculator

1 B is not correct. 120/o chose th is.


2 Am iodarone is used as short-term therapy in adults w ith a life-threatening wide-complex tachycardia such as ventricular
3 tachycard ia or ventricula r fibrillation . Am iodarone is an effective therapeutic option with supraventricu lar tachycardia (SVT) .
4 However, it is reserved for patients who have failed other therapies due to its significant toxicity, including fatal interstit ial
lung disease, hyper- and hypo-thyroid ism, and liver damage.
5 Amiodarone Supraventricular tachycardia Interstitial lung disease Ventricular fibrillation Ventricular tachycardia Tachycardia liver Respiratory disease lung Fibrillation Hepatotoxicity
6 Toxicity
7
•8
C i s not co rr ect. 150/o chose this .
Metoprolol, a ~-b l ocker, slows conduction through the atrioventricu lar (AV) node and is a good choice for short- or long-term
•9
management of paroxysmal supraventricu lar tachycard ia . However, a patient with a Wolff-Parkinson-White arrhythm ia shou ld
• 10 avoid AV nodal slowing agents as these may make the arrhythmia worse. Th is patient has a known anatomic abnormality
• 11 classically associated with supraventricular tachycardia and requires a more permanent solution .
Supraventricular tachycardia ~1etoprolol Atrioventricular node Cardiac arrhythmia Tachycardia Paroxysmal attack
• 12
• 13 D i s not correct. 100/o chose this .
• 14 Verapam il , a calcium channel blocker, slows conduction t h rough the heart and is a good choice for short- or long-term
• 15 management of paroxysmal supraventricu lar tachycardia (SVT) . However, this patient has a known anatomic abnormality
classically associated with SVT and requires a more permanent solution .
• 16
Calcium channel blocker Supraventricular tachycardia Verapamil Tachycardia Calcium channel Calcium
• 17
E i s not correct. S O/o c hose th is .
• 18
A vaga l maneuver (blocking atrioventricu lar noda l conduction) is the preferred initial therapy to terminate acute episodes of
• 19
supraventricular tachycardia (SVT) . Vagal maneuvers include Valsalva and application of ice to the face. However, this patient
• 20 has a known anatomic abnormality classica lly associated with SVT and requ ires a more permanent solution .
• 21 Supraventricular tachycardia Vagal maneuver Tachycardia Vagus nerve

• 22 F is not correct. 6 0/o chose this .


• 23 Electrical cardioversion is necessary for hemodynam ica lly unstable patients w ith arrhythmias refractory to more conservative
• 24

Lock
s
Suspend
0
End Block
Item: 7 of 24 ~ 1 • Mark -<] C> Jill ~· ~J
QID: 23646 ..1 Previous Next Lab'V!I!ues Notes Calculator

1 • classically associated with supraventricular tachycardia and requires a more permanent solution .
Supraventricular tachycardia Metoprolol Atrioventricular node Cardiac arrhythmia Tachycardia Paroxysmal attack
2
3 D is not correct. 100/o chose this.
4 Verapam il , a calcium channel blocker, slows conduction t h rough the heart and is a good choice for short- or long-term
5
management of paroxysmal supraventricular tachycardia (SVT) . However, this patient has a known anatomic abnormality
classically associated with SVT and requires a more permanent solution .
6 Calcium channel blocker Supraventricular tachycardia Verapamil Tachycardia Calcium channel Calcium
7
•8
E is not correct. SO/o chose this .
A vaga l maneuver (blocking atrioventricu lar noda l conduction) is the preferred initial therapy to terminate acute episodes of
•9
supraventricular tachycardia (SVT) . Vagal maneuvers include Valsalva and appl ication of ice to the face. However, this patient
• 10 has a known anatomic abnormality classica lly associated with SVT and requires a more permanent solution .
• 11 Supraventricular tachycardia Vagal maneuver Tachycardia Vagus nerve

• 12 F is not correct. 60/o chose this .


• 13 Electrical cardioversion is necessary for hemodynam ica lly unstable patients w ith arrhythmias refractory to more conservative
• 14 management .
Cardioversion Cardiac arrhythmia Hemodynamics Electrical cardioversion
• 15
• 16
• 17 Bottom line:
• 18
Radiof requency ablation of the pathways responsible for a supraventricu lar tachycardia (SVT) due to Ebstein anomaly is the
• 19 preferred definitive treatment .
Supraventricular tachycardia Radiofrequency ablation Tachycardia Radio frequency Ablation
• 20
• 21
• 22
References:
• 23
FA Step 2 CK 8th ed pp 289; 356
• 24

Lock
s
Suspend
0
End Block
Item: 8 of 24 ~ 1 • Mark -<] C> Jill ~· ~J
QID: 21586 ..1 Previous Next Lab'V!I!ues Notes Calculator

1 •
A girl is delivered at 34 weeks' gestation v ia spontaneous vaginal delivery. She is the product of a pregnancy that was
2 complicated by preterm labor. Prenatally the mother was blood type A and was rubella immune and negative for Rh
3 antibody, Group B streptococci, rapid plasma reagin, hepatitis B surface antigen, gonorrhea, and chlamydia. There is no
4 mecon ium. Her birth weight is 2 kg ( 4 lb 8 oz) . Four days after birth, she develops grunting, nasal flaring, and intercosta l
retractions. She is acyanotic. Her respiratory rate is 70/min, and she does not respond to administration of blow-by oxygen.
5
Cardiac auscultation reveals a continuous machinery murmur heard at the left upper sternal border. A prominent apical impu lse
6 is also palpable . Physical exam reveals bounding peripheral pulses bilaterally and rooting reflexes . X- ray of the chest shows
7 cardiomegaly and increased pulmonary vascular markings. Echocardiography does not show any septal defects or valvular
•8 abnormalities .
•9
• 10
What is the most appropriate next step in management of this patient?
• 11

• 12 A. Balloon atria l septostomy


• 13 B. Coil embolization
• 14
C. Indomethacin
• 15
• 16 D. Prostaglandin E
• 17 E. Surgical ligation
• 18
• 19
• 20
• 21
• 22
• 23
• 24

Lock
s
Suspend
0
End Block
Item: 8 of 24 ~ 1 • Mark -<] C> Jill ~· ~J
QID: 21586 ..1 Previous Next Lab'V!I!ues Notes Calculator

2
3 The correct answer i s C. 620/o chose this .
4 This patient has respiratory distress and signs of congestive heart failure, with no response to supplemental oxygen . This
5
presentation indicates a congenita l heart defect, wh ich should be evaluated by echocardiography. The most likely defect is a
patent ductus arteriosus (PDA), which is a vascular connection that exists between the aorta and the main pulmonary artery.
6 It is more common in preterm infants and accounts for 10% of congenital heart disease. Small PDAs are asymptomatic, wh ile
7 large ones may cause congestive heart failure, fail ure to thrive, and recurrent lower respiratory tract infections . On physical
8 examination, a continuous machinery murmur heard best at the left upper sternal border and left infraclavicular region is
•9
present. There may also be a prominent apical impu lse a1nd a thrill. X-ray of the chest may show card iomegaly and increased
pulmonary vascular markings. Although PDAs usually close with in the first month of life, in preterm infants with clinically
• 10 sign ificant PDAs, cyclooxygenase inhibitors ( eg, ibuprofen, indomethacin) are the preferred treatment. These agents act by
• 11 inhibit ing the synthesis of prostaglandin E2 , wh ich is involved in maintaining ductal patency.
Ibuprofen Patent ductus arteriosus Indometacin Cardiomegaly Congenital heart defect Prostaglandin Heart failure Congestive heart failure Pulmonary artery Chest radiograph
• 12
Echocardiography Aorta Cyclooxygenase Failure to thrive Ductus arteriosus Asymptomatic Blood vessel Cardiovascular disease Dyspnea X-ray Oxygen Congenital disorder
• 13
• 14 Pretenm birth Physical examination

• 15 A i s not correct. 3% ch ose this .


• 16 Balloon atria l septostomy is used to create a connection between the atria in transposition of the great arteries (TGA), to
• 17 allow mixing of deoxygenated and oxygenated blood for patient survival. This patient's continuous machinery-li ke murmur is
• 18
not consistent with TGA. TGA itself is not typically associated with a murmur, but instead commonly presents with murmurs
due to other associated abnormalities such as ventricular septal defect (VSD), in wh ich a pansystolic murmur wou ld be heard
• 19 at the left lower sternal border.
• 20 Ventricular septal defect Transposition of the great vessels Heart munmur Atrium (heart) Systole Artery Great arteries Sternum Blood Ventricular system

• 21
B is not correct. 2 0/o chose this .
• 22
Coil embolization is used in the setting of a patent ductus arteriosus (PDA) that fails to close w ith medical therapy. It is not a
• 23 part of the initial management of PDA in preterm infants .
• 24 Patent ductus arteriosus Ductus arteriosus Patent

Lock
s
Suspend
0
End Block
Item: 8 of 24 ~ 1 • Mark -<] C> Jill ~· ~J
QID: 21586 ..1 Previous Next Lab'V!I!ues Notes Calculator
:T• I. •t.t•• . II I I •t•.T•" -·.I J • J ~ "I"'"" • •• •• • I ~lilT" I. T•
1

2 B is not correct. 20/o chose this.


3 Coil embolization is used in the setting of a patent ductus arteriosus (PDA) that fails to close w ith medical therapy. It is not a
4
part of the init ial management of PDA in preterm infants.
Patent ductus arteriosus Ductus arteriosus Patent
5
6 D is not correct. 300/o chose this.
7
Prostaglandin E is used to maintain a patent ductus arteriosus ( PDA). It is used in the setting of congenital heart defects that
require the pr esence of a PDA for survival, such as t ransposition of the great arteries, hypoplastic left heart, critical aortic or
8
pulmonic stenosis, crit ical coarctation of the aorta, and tetralogy of Fallot (with pulmonary atresia ) .
•9 Tetralogy of Fallot Patent ductus arteriosus Coarctation of the aorta Prostaglandin Pulmonic stenosis Transposition of the great vessels Pulmonary atresia Aorta

• 10 Pulmonary valve stenosis Ductus arteriosus Congenital heart defect Congenital disor der Stenosis Atresia Great arteries Hypoplasia Heart Artery

• 11
E is not correct. 30/o chose this .
• 12
Surgical ligation is used in the setting of a patent ductus arteriosus ( PDA) that fails to close with medical therapy. It is not a
• 13 part of the init ial management of PDA in preterm infants.
• 14 Patent ductus arteriosus Ugature (medicine) Ductus arteriosus Patent

• 15
• 16
Bottom line:
• 17
PDA is a vascu lar connection that exists between the aorta and the main pulmonary artery. In preterm infants,
• 18
cyclooxygenase inhibitor s (eg, ibuprofen , indomethacin) are the init ial treatment used to close clinically significant PDAs.
• 19 Ibuprofen Indometacin Cyclooxygenase Pulmonary artery Aorta Blood vessel

• 20
• 21
• 22 References:
FA Step 2 CK 9th ed p 375
• 23
FA Step 2 CK 8th ed pp 357-358
• 24

Lock
s
Suspend
0
End Block
Item: 9 of 24 ~ 1 • Mark -<] C> Jill ~· ~J
QID: 237 20 ..1 Previous Next Lab'V!I!ues Notes Calculator

1 •
A 7-year-old boy presents to the pediatrics outpatient cli nic for the first t ime. His mother reports that he has r eceived all ~~AI
2 his normal vaccinations. He is at normal weight and height and appears to be developing normally. He has no significant
3 medical history, but physical examination r evea ls a harsh systolic crescendo-decrescendo murmur at the second left
4 intercostal space with radiation into the neck . The physician suspects pulmonary valvular stenosis because the murmur was
heard loudest in the pu lmonic region .
5
6
Which of the following changes in the quality of the murmur will occur with the listed maneuver if the presumptive diagnosis is
7
correct"
8
•9
A. Decrease in intensity w ith inhalation of amyl nit rail:e
• 10
• 11
B. Increase in intensity with change in posit ion from supine or squatting to standing
• 12 C. Increase in intensity with handgrip
• 13
D. Increase in intensity with inspiration
• 14
• 15
E. Increase in intensity with Valsalva maneuvers
• 16
• 17
• 18
• 19
• 20
• 21
• 22
• 23
• 24

Lock
s
Suspend
0
End Block
Item: 9 of 24 ~ 1 • Mark -<] C> Jill ~· ~J
QID: 237 20 ..1 Previous Next Lab'V!I!ues Notes Calculator

2 The correct answer is D. 540/o chose this.


3 The murmur of pulmonary valvular stenosis is described as a harsh systolic crescendo-decrescendo murmur that is heard
4 best at the second left intercosta l space. With severe pulmonary stenosis, radiation of the murmur into the left neck is
sometimes noted. I n contrast, the murmur of aortic stenosis is described as a mid-systolic ejection murmur heard best at the
5
right second intercostal space with radiat ion into the bilatera l carotid arteries. Mitral regurgitation and tricuspid regu rgitation
6 both present with holosystolic murmurs, with the murm ur of mitral regurgitation heard best at the apex and the murmur of
7 tricuspid regurgitation heard best at the left lower sternal border. As the various systolic murmurs can be difficult to
8 differentiate, a number of maneuvers can aid in diagnosis. With inspiration, venous retu rn to the right ventricle increases,
causing an increase in volume through the right side of the heart. Inspiration increases the intensity of both systolic
9
(pulmonary stenosis, tricuspid regurgitation) and diastolic (pulmonary regurgitation, tricuspid stenosis) right-sided murmurs .
• 10 Left-sided murmurs either do not change or decrease in intensity with inspiration . While physical examination is important in
• 11 the diagnosis of valvular disorders, ultimate diagnosis depends on the echocardiogram .
Tricuspid valve stenosis Pulmonary insufficiency Systole Aortic stenosis Pulmonic stenosis Intercostal space Common carotid artery Ventricle (heart) Carotid artery Mitral insufficiency
• 12
Tricuspid insufficiency Echocardiography Diastole Stenosis Heart murmur Artery Systolic heart murmur Tricuspid valve Regurgitation (circulation) Heart Physical examination Sternum
• 13
• 14 Vein Tricuspid atresia

• 15 A is not correct. 9% chose this .


• 16 Inhalation of amyl nitrate is infrequently used today but used to be done at the bedside for murmur evaluation . Amyl nitrate
• 17 is an arteriolar dilator that causes vasodilation, decrease in systemic vascular resistance, decrease in arterial pressure, and
an overall decrease in the left ventricular volume . Inhalation of amyl nitrate causes decrease in murmurs of mitral
• 18
regurgitation, ventricula r septal defect, and tetralogy of Fallot. Murmurs of aortic stenosis, pulmonary stenosis, and
• 19 hypertrophic obstructive cardiomyopathy are increased .
• 20 Tetralogy of Fallot Ventricular septal defect Aortic stenosis Vasodilation Hypertrophic cardiomyopathy Mitral insufficiency Pulmonic stenosis Stenosis Cardiomyopathy Vascular resistance

• 21 Hypertrophy Regurgitation (circulation) Blood vessel Nitrate Heart murmur Ventricu lar hypertrophy Arteriole Blood pressure Ventricle (heart) Heart

• 22
B is not correct. 110/o chose this .
• 23
A change in position from supine or squatting to a standing position causes a decrease in venous retu rn to the heart. As a
• 24 ... ---···"' ....._ ___ =-- -'------- :- -:-1-.&.. __ _. •-.c.&.. ··--""'-:-.• •-- ..J:--.&.-1:- ·· - ·· · - - - _, _______ =- -"'--' ·- ··- ···-- _, _______ :- -...&.- ...: - •

Lock
s
Suspend
0
End Block
Item: 9 of 24 ~ 1 • Mark -<] C> Jill ~· ~J
QID: 237 20 ..1 Previous Next Lab'V!I!ues Notes Calculator
. ~ :. :,- . ;.: . :t :,; -
1

2 B is not correct. 110/o chose this.


3 A change in position from supine or squatting to a standing position causes a decrease in venous return to the heart. As a
4 result, there is a decrease in right and left ventricular diastolic volumes, decrease in stroke volume, decrease in arteria l
pressu re, and reflex increase in heart rate . This decreases the murmurs of pulmonary stenosis, aortic stenosis, mitral
5
regurgitation, tricuspid regurgitation, and ventricular septal defect. The on ly murmur that increases in intensity w ith standing
6 is the systolic ejection murmur of hypertrophic obstructive cardiomyopathy (HOCM) . The murmur of HOCM increases as a
7 result of increased obstruction to the aortic outflow tract . This is often a useful technique for diagnosing HOCM .
Ventricular septal defect Aortic stenosis Hypertrophic cardiomyopathy Stroke volume Mitral insufficiency Pulmonic stenosis Systole Diastole Stenosis Hypertrophy Tricuspid insufficiency
8
Regurgitation (circulation) Ventricular hypertrophy cardiomyopathy Tricuspid valve Tricuspid atresia Heart rate Vein Stroke Heart munmur Supine position Heart Blood pressure
9
• 10 C is not correct. 150/o chose this .
• 11 Sustained handgrip increases systemic vascular resistance, arterial pressure, cardiac output, left ventricular volume, and
• 12 filling pressu re. The handgrip maneuver is most useful in differentiating the systolic ejection murmur of aortic stenosis and
the regurgitant murmur of mitral insufficiency. The murmur of aortic stenosis decreases in intensity wh ile the murmur of
• 13
mitral regurgitation increases in intensity with handgrip. The murmurs of aortic regurg itation, mitral stenosis, and ventricular
• 14 septal defect increase in intensity, while the murmur of hypertrophic obstructive cardiomyopathy decreases in intensity. The
• 15 murmur of pu lmonary stenosis is not affected .
Ventricular septal defect Aortic insufficiency Cardiac output Hypertrophic cardiomyopathy Pulmonic stenosis Aortic stenosis Stenosis Mitral valve stenosis Vascular resistance Systole
• 16
Regurgitation (circulation) Mitral insufficiency Hypertrophy Handgrip maneuver Ventricular hypertrophy Cardiomyopathy Blood vessel Heart Blood pressure Heart munmur
• 17
• 18 E is not correct. 11% chose this.
• 19 With the Valsalva maneuver, there is a decrease in venous retu rn , right and left ventricular volumes, stroke volume, and a
• 20 decrease in arterial and pulse pressure. As a resu lt, all flow murmurs decrease (aortic stenosis, pu lmonary stenosis, mitral
regurgitation, tricuspid regurgitation, aortic regurgitation, pulmonary regurgitation, mitral stenosis, and tricuspid stenosis) .
• 21
The on ly murmur that increases in intensity is the murmu r of hypertrophic cardiomyopathy.
• 22 Functional munmur Tricuspid valve stenosis Valsalva maneuver Hypertrophic cardiomyopathy Pulmonary insufficiency Aortic insufficiency Aortic stenosis Mitral valve stenosis

• 23 Stroke volume Pulmonic stenosis Pulse pressure Cardiomyopathy Mitral insufficiency Tricuspid insufficiency Stenosis Regurgitation (circulation) Ventricular hypertrophy Hypertrophy
• 24 y

Lock
s
Suspend
0
End Block
Item: 9 of 24 ~ 1 • Mark -<] C> Jill ~· ~J
QID: 237 20 ..1 Previous Next Lab'V!I!ues Notes Calculator

• • • • •
1
Sustained handgrip increases systemic vascular resistance, arterial pressure, cardiac output, left ventricular volume, and
2
filling pressure . The handgrip maneuver is most useful in differentiating the systolic ejection murmur of aortic stenosis and
3 the regurgitant murmur of mitral insufficiency. The murmur of aortic stenosis decreases in intensity wh ile the murmur of
4 mitral regurgitation increases in intensity with handgrip. The murmurs of aortic regurgitation, mitral stenosis, and ventricular
septal defect increase in intensity, while the murmur of hypertrophic obstructive cardiomyopathy decreases in intensity. The
5
murmur of pulmonary stenosis is not affected .
6 Ventricular septal defect Aortic insufficiency Cardiac output Hypertrophic cardiomyopathy Pulmonic stenosis Aortic stenosis Stenosis Mitral valve stenosis Vascular resistance Systole
7 Regurgitation (circulation) Mitral insufficiency Hypertrophy Handgrip maneuver Ventricular hypertrophy Cardiomyopathy Blood vessel Heart Blood pressure Heart murmur
8
E is not correct. 11% chose this.
9
With the Valsalva maneuver, there is a decrease in venous return , right and left ventricular volumes, stroke volume, and a
• 10
decrease in arterial and pulse pressure. As a result, all flow murmurs decrease (aortic stenosis, pulmonary stenosis, mitral
• 11 regurgitation, tricuspid regurgitation, aortic regurgitation, pulmonary regurgitation, mitral stenosis, and tricuspid stenosis) .
• 12 The only murmur that increases in intensity is the murmu r of hypertrophic cardiomyopathy.
Functional murmur Tricuspid valve stenosis Valsalva maneuver Hypertrophic cardiomyopathy Pulmonary insufficiency Aortic insufficiency Aortic stenosis Mitral valve stenosis
• 13
Stroke volume Pulmonic stenosis Pulse pressure Cardiomyopathy Mitral insufficiency Tricuspid insufficiency Stenosis Regurgitation (circulation) Ventricular hypertrophy Hypertrophy
• 14
Tricuspid valve Stroke Heart Vein Tricuspid atresia Heart murmur
• 15
• 16
• 17 Bottom line:
• 18
Inspiration increases the intensity of both systolic (pulmonary stenosis, tricuspid regurgitation) and diastolic (pulmonary
• 19 regurgitation, tricuspid stenosis) right-sided murmurs. During inspiration, there is an increase in the venous return to the
• 20 right ventricle, wh ich is the reason for the increase in the intensity of the murmur.
Tricuspid valve stenosis Pulmonary insufficiency Pulmonic stenosis Tricuspid insufficiency Ventride (heart) Systole Diastole Stenosis Regurgitation (circulation) Tricuspid valve
• 21
Tricuspid atresia Heart murmur
• 22
• 23
• 24

Lock Suspend
s 0
End Block
Item: 10 of 24 ~ 1 • Mark <:] (:>- Jill ~· ~J
QID: 21548 ..1 Previous Next Lab'V!I!ues Notes Calculator

1 •
A 12-year-old girl is brought to the emergency department when she loses consciousness after a game of tag with her
2 younger siblings. Her mother reports a similar episode last week after a game of soccer. When the patient was 1 year old,
3 she had a murmur that was not worked up. On physical examination the patient appears small for her age. Her
4 temperature is 37°C (98 .6°F), pulse is 80/ min, blood pressure is 110/ 60 mm Hg, and r espiratory rate is 20/ min . She is diffusely
cyanotic and has clubbing of her digits bilatera lly. There is a palpable right ventricular impulse and a short systolic ejection
5
murmur loudest at the left lower sternal bor der. There is also a loud ej ection sound and a closely split 52, w ith a significant
6 pulmonic component audible at the second left intercostal space . Her lung sounds are diminished bilaterally. X-ray of the chest
7 shows right heart en largement, dilated main pulmonary a1rteries, and attenuated peripheral pulmonary arteries.
8
9 Which of the following is the most likely diagnosis?
• 10
• 11 A. Atrial septal defect with left-to- right shunt
• 12
B. Coarctation of the aorta
• 13
• 14
C. Tetralogy of Fallot

• 15 D. Ventricular septal defect w ith left-to- right shunt


• 16
E. Ventricular septal defect w ith ri ght-to-left shunt
• 17
• 18
• 19
• 20
• 21
• 22
• 23
• 24

Lock
s
Suspend
0
End Block
Item: 10 of 24 ~ 1 • Mark <:] (:>- Jill ~· ~J
QID: 21548 ..1 Previous Next Lab'V!I!ues Notes Calculator

2 The correct answer is E. 40% chose this.


3 The patient's presentation and radiographic findings are consistent with pulmonary circulation overload due to a congen ital
4 left-to-right shunt. Chronic overload of pu lmonary circu lation leads to irreversible pulmonary vascular inj ury and increased
5
pulmonary vascular resistance. Eventually, the pulmonary resistance becomes higher than the systemic resistance, and the
shunt switches directions to become a right-to-left shunt . This triad of pathologic commun ication between systemic and
6 pulmonary circulations, pulmonary arterial disease, and cyanosis is called Eisenmenger syndrome .
7
Eisenmenger syndrome is a congen ital disease in which chronic left-to-right shunting leads to increased pu lmonary blood flow
8 and increased pu lmonary vascular resistance. This results in reversal of the left-to-right shunt to form a right-to-left shunt,
9 causing cyanosis to develop . It can manifest with fatigue, chest pain, shortness of breath, dyspnea on exertion, and
10 exertional syncope. On examination, central cyanosis and digital clubbing are common. A short systolic murmur, a loud
ej ection sound, and a single or closely split 52 with a prominent pulmon ic component may also be present. Signs of right
• 11
heart overload are also often present, includ ing a palpable right ventricu lar impulse and edema .
• 12
Eisenmenger's syndrome Cyanosis Dyspnea Pulmonary circulation Nail clubbing Right-to-left shunt Syncope (medicine) Split 52 Heart Vascular resistance Systole Edema
• 13
Cardiac shunt Congenital disorder Shunt (medical) Blood vessel Chest pain Ventricle (heart) Heart murmur Fatigue (medical)
• 14

• 15 A is not correct. 110/o chose this.


• 16 Atrial septal defect (ASD) is a congenita l cardiac abnormality that is usually discovered incidentally. The primum ASDs resu lt
from an aberrant development of endocard ial cushions and may be associated with other defects including m itral valve and
• 17
atrioventricular canal defects. The secundum ASD resu lts from an excessive resorption of primum septum or arrested growth
• 18 of secundum septum and is not associated w ith other cardiac defects . Most ASDs are the secundum type . The natural history
• 19 of an ASD depends on the size. While smaller defects <Smm in diameter tend to close spontaneously, moderate to large
• 20
defects may lead to increasing left-to-right shunting. The right-sided volume overload is exceptionally well tolerated and
patients generally do not develop symptoms before 40 years of age. Presenting symptoms are most frequently atrial
• 21
arrhythmias leading to palpitations, exercise intolerance, dyspnea, and fatigue. Pulmonary vascu lar disease occurs in only
• 22 10% of adults with isolated ASDs but is rare in children or adolescents .
• 23 Atrial septal defect Dyspnea Endocardial cushions Mitral valve Congenital disorder Atrial fibrillation cardiac arrhythmia Septum Endocardium Volume overload Symptom Atrium (heart)

P::~lnit::.tion~
• 24

Lock
s
Suspend
0
End Block
Item: 10 of 24 ~ 1 • Mark <:] (:>- Jill ~· ~J
QID: 21548 ..1 Previous Next Lab'V!I!ues Notes Calculator

1 A i s not correct. 110/o chose this.


2 Atrial septal defect (A5D) is a congenita l cardiac abnormality that is usually discovered incidentally. The primum A5Ds resu lt
3 from an aberrant development of endocardial cushions and may be associated with other defects including mitral valve and
4
atrioventricular canal defects. The secundum A5D results from an excessive resorption of primum septum or arrested growth
of secundum septum and is not associated w ith other cardiac defects . Most A5Ds are the secundum type . The natural history
5 of an A5D depends on the size. While smaller defects <Smm in diameter tend to close spontaneously, moderate to large
6 defects may lead to increasing left-to-right shunting. The right-sided volume overload is exceptionally well tolerated and
7 patients generally do not develop symptoms before 40 years of age. Presenting symptoms are most frequently atrial
arrhythmias leading to palpitations, exercise intolerance, dyspnea, and fatigue. Pulmonary vascu lar disease occurs in only
8
10% of adults with isolated A5Ds but is rare in children or adolescents.
9 Atrial septal defect Dyspnea Endocardial cushions Mitral valve Congenital disorder Atrial fibrillation Cardiac arrhythmia Septum Endocardium Volume overload Symptom Atrium (heart)
10 Palpitations
• 11
B is not correct. SO/o chose this .
• 12
Coarctation of the aorta is not commonly associated with Eisenmenger syndrome . It manifests with a systolic ejection
• 13
murmur at the left upper sternal border, radiating to the interscapular region . Physical exam ination is also remarkable for
• 14 weak and delayed femoral pulses relative to the upper extremity pulses and hypertension in the upper extremit ies . Heart
• 15 failure rarely occurs beyond the neonatal period .
Eisenmenger's syndrome Coarctation of the aorta Aorta Hypertension Systole Sternum Heart failure Physical examination Femur Infant Heart murmur Stenosis
• 16
• 17 C i s not correct. 2 6 0/o chose this .
• 18 Tetralogy of Fallot (TOF) usually presents with cyanotic spells in infancy. During development, there is anterior and cephalad
• 19
deviation of the infundibular septum . This results in a malaligned ventricular septal defect (V5D), with the aortic root
overriding the defect and leading to subsequent right ventricular outflow obstruction . The ensuing right ventricular
• 20 hypertrophy is thought to be a response to the large V5D and right ventricular outflow obstruction . On examination, there is
• 21 a normal 51, single 52, and a harsh systolic crescendo-decrescendo ejection murmur heard best over the left upper sternal
• 22 border, which radiates to the back. It is also possible to have a holosystolic murmur at the left lower sternal border from the
V5D, but the murmur in TOF is due mainly to the right ventricular outflow obstruction . A continuous machinery murmur may
• 23
also be heard if a PDA is present.
• 24

Lock
s
Suspend
0
End Block
Item: 10 of 24 ~ 1 • Mark <:] (:>- Jill ~· ~J
QID: 21548 ..1 Previous Next Lab'V!I!ues Notes Calculator

1 C is not correct. 260/o chose this.


2 Tetralogy of Fallot (TOF) usually presents with cyanotic spells in infancy. During development, there is anterior and cephalad
3 deviation of the infund ibular septum. This results in a malaligned ventricular septal defect (VSD), with the aortic root
4
overriding the defect and leading to subsequent right ventricular outflow obstruction . The ensuing right ventricular
hypertrophy is thought to be a response to the large VSD and right ventricular outflow obstruction . On examination, there is
5 a normal 51, single 52, and a harsh systolic crescendo-decrescendo ejection murmu r hea rd best over the left upper sternal
6 border, wh ich radiates to the back. It is also possible to have a holosystolic murmur at the left lower sternal border from the
7 VSD, but the murmu r in TOF is due mainly to the right ventricular outflow obstruction. A continuous machinery murmur may
also be heard if a PDA is present.
8
9
The clinical presentation of TOF is dependent upon the degree of right ventricular outflow obstruction. Severe obstruction
leads to inadequate pu lmonary flow which presents in the immediate newborn period with profound cyanosis. Children with
10
moderate obstruction may present w ith hypercyanotic "tet" spells when right ventricular outflow t ract is obstructed during
• 11 periods of ag itation . Children with minimal obstruction may present with pulmonary overci rculation from right-to-left shunt
• 12 through the VSD and ensuing heart failure as in this patient. However, this is not the most likely diagnosis .
• 13 Tetralogy of Fallot Ventricular septal defect Cyanosis Right-to-left shunt Right ventricular hypertrophy Cyanotic heart defect Systole Ventricular hypertrophy Aorta Hyoertrophy

• 14 Heart failure Shunt (medical) Ascending aorta Septum Heart murmur Infundibulum (heart) Ventricular outflow tract Sternum

• 15
D is not correct. 180/o chose this .
• 16
Ventricular septal defects (VSDs) can occu r in the membranous portion or muscula r portion . The location does not affect the
• 17 volume of shunt, but does determine the likeli hood of closure. Muscula r defects often close spontaneously. VSDs initially lead
• 18 to a left-to-right shunt with blood flowing from the high pressure left ventricle to the lower pressure system of the right
• 19
ventricle. Small defects are usually ben ign, but larger defects lead to a significant overload of pulmonary circulation, which
leads to pulmonary vascular damage and pulmonary hypertension over time .
• 20
Patients with a small VSD generally remain asymptomatic into adulthood, while those w ith a large VSD present with heart
• 21
failure during infancy or develop Eisenmenger syndrome, which presents during late childhood to early adulthood .
• 22 Eisenmenger syndrome is a congen ital disease in which chronic left-to-right shunting leads to increased pu lmonary blood flow
• 23 and increased pu lmonary vascular resistance. This results in reversal of the left-to-right shunt to form a right-to-left shunt,
• 24 causina cvanosis to develoo . It can manifest with fatiaue, chest oain, shortness of breath, dvsonea on exertion, and

Lock
s
Suspend
0
End Block
Item: 10 of 24 ~ 1 • Mark <:] (:>- Jill ~· ~J
QID: 21548 ..1 Previous Next Lab'V!I!ues Notes Calculator

1 • Tetralogy of Fallot Ventricular septal defect Cyanosis Right-to-left shunt Right ventricular hypertrophy Cyanotic heart defect Systole Ventricular hypertrophy Aorta Hypertrophy
2 Heart failure Shunt (medical) Ascending aorta Septum Heart murmur Infundibulum {heart) Ventricular outflow tract Sternum
3
D is not correct. 180/o chose this .
4
Ventricular septal defects (VSDs) can occu r in the memb ranous portion or muscula r portion . The location does not affect the
5
volume of shunt, but does determine the likeli hood of closure. Muscular defects often close spontaneously. VSDs init ially lead
6 to a left-to-right shunt with blood flowing from the high pressure left ventricle to the lower pressure system of the right
7 ventricle. Small defects are usually ben ign, but larger defects lead to a significant overload of pulmonary circulation, which
leads to pulmonary vascular damage and pulmonary hypertension over t ime.
8
9 Patients with a small VSD generally remain asymptomatic into adulthood, while those with a large VSD present with heart
failure during infancy or develop Eisenmenger synd rome, which presents during late childhood to early adulthood .
10
Eisenmenger syndrome is a congen ital disease in which chronic left-to-right shunting leads to increased pu lmonary blood flow
• 11 and increased pu lmonary vascular resistance. This results in reversal of the left-to-right shunt to form a right-to-left shunt,
• 12 causing cyanosis to develop . It can manifest with fatigue, chest pain, shortness of breath, dyspnea on exertion, and
• 13 exertional syncope. On examination, central cyanosis and digital clubbing are common . A short systolic mu rmur, a loud
ej ection sound, and a single or closely split 52 with a prominent pulmon ic component may also be present. Signs of right
• 14
heart overload are also often present, including a palpable right ventricu lar impulse and edema .
• 15
Eisenmenger's syndrome Cyanosis Dyspnea Pulmonary circulation Pulmonary hypertension Nail dubbing Right-to-left shunt Ventricle (heart) Syncope (medicine) Hypertension Systole
• 16
Vascular resistance Split 52 Heart Congenital disorder Interventricular septum Heart failure Edema Asymptomatic Shunt (medical) Cardiac shunt Fatigue (medical) Chest pain
• 17
Blood vessel Congenital heart defect Heart murmur
• 18
• 19
• 20 Bottom line :
• 21 Eisenmenger syndrome is a congen ital disease in which chronic left-to-right shunting leads to increased pu lmonary blood
• 22 flow with increased pu lmonary vascular resistance, which ult imately leads to a right-to-left shunt reversal.
Eisenmenger's syndrome Congenital disorder Vascular resistance Right-to-left shunt Shunt (medical)
• 23
• 24

Lock
s
Suspend
0
End Block
Item: 10 of 24 ~ 1 • Mark <:] (:>- Jill ~· ~J
QID: 21548 ..1 Previous Next Lab'V!I!ues Notes Calculator

1 Ventricular septal defects {VSDs) can occur in the membranous portion or muscular portion . The location does not affect the
2
volume of shunt, but does determine the likelihood of closure. Muscular defects often close spontaneously. VSDs initially lead
to a left-to-right shunt with blood flowing from the high pressure left ventricle to the lower pressure system of the right
3
ventricle. Small defects are usually benign, but larger defects lead to a significant overload of pu lmonary circulation, wh ich
4 leads to pulmonary vascular damage and pulmonary hypertension over time.
5 Patients with a small VSD generally remain asymptomatic into adulthood, while those w ith a large VSD present with heart
6 failure during infancy or develop Eisenmenger syndrome, wh ich presents during late childhood to early adulthood .
7 Eisenmenger syndrome is a congenital disease in which chronic left-to-right shunting leads to increased pulmonary blood flow
and increased pulmonary vascular resistance. This results in reversal of the left-to-right shunt to form a right-to-left shunt,
8
causing cyanosis to develop . It can manifest with fatigue, chest pain, shortness of breath, dyspnea on exertion, and
9 exertional syncope. On examination, central cyanosis and digital clubbing are common . A short systolic murmur, a loud
10 ejection sound, and a single or closely split 52 with a prominent pulmonic component may also be present. Signs of right
• 11
heart overload are also often present, includ ing a palpab le right ventricular impulse and edema .
• 12 Eisenmenger's syndrome Cyanosis Dyspnea Pulmonary circulation Pulmonary hypertension Nail clubbing Right-to-left shunt Ventricle (heart) Syncope (medicine) Hypertension Systole

• 13 Vascular resistance Split 52 Heart Congenital disorder Interventricular septum Heart failure Edema Asymptomatic Shunt (medical) Cardiac shunt Fatigue (medical) Chest cain

• 14 Blood vessel Congenital heart defect Heart murmur

• 15
• 16
Bottom line:
• 17
Eisenmenger syndrome is a congenital disease in which chronic left-to-right shunting leads to increased pulmonary blood
• 18
flow with increased pulmonary vascular resistance, wh ich ultimately leads to a right-to-left shunt reversal.
• 19 Eisenmenger's syndrome Congenital disorder Vascular resistance Right-to-left shunt Shunt (medical)

• 20
• 21
• 22 References:
FA Step 2 CK 9th ed pp 371-378
• 23
FA Step 2 CK 8th ed pp 355-360
• 24

Lock
s
Suspend
0
End Block
Item: 11 of 24 ~ 1 • Mark <:] (:>- Jill ~· ~J
QID: 23397 ..1 Previous Next Lab'V!I!ues Notes Calculator

1 •
A 5-year-old boy presents to the emergency department with a history of 1 week of fever. On physical examination, the ~~AI
2 patient has bilateral conjunctivit is, cervical lymphadenopathy, extremity edema , and oral mucosal changes. The rash is
3 mainly on the trunk, is erythematous, and polymorphous.
4
:------
5
6
7
8
9
10
· 11
• 12
• 13
• 14
• 15
• 16 Which of the following is a serious complication of this disease?
• 17
• 18 A. Aortic stenosis
• 19
B. Cor onary aneurysm
• 20
• 21 C. Mitral valve pr olapse
• 22 D. Myocardial infarction
• 23
E. Pulmonary hypertension
• 24

Lock
s
Suspend
0
End Block
Item: 11 of 24
QIO: 23397

2
~
~
p
1 • Mark

.
<::::J
PreVIOUS

J
t::>
Next

I
iii
Lab 'i?.llues

y p p
mainly on the trunk, is erythematous, and polymorphous.
9'
.
Notes
~:·11
Calculator

y, y . . g ...
3
4

5
6
7
8
9
10
· 11
• 12
• 13
• 14
Which of the following is a serious complication of this disease?
• 15
• 16
A. Aortic stenosis
• 17
• 18 B. Coronary aneurysm
• 19
C. Mitral valve prolapse
• 20
D. Myocardial infarction
• 21
• 22 E. Pulmonary hypertension
• 23

• 24

a
Lock
s
Suspend
8
End Block
Item: 11 of 24 ~ 1 • Mark <:] (:>- Jill ~· ~J
QID: 23397 ..1 Previous Next Lab'V!I!ues Notes Calculator

2 The correct answer is B. 790/o chose th is.


3 This patient has Kawasaki's disease . Criteria for diagnosis include at least 5 days of fever (usually >40°C) plus fou r of five of
4
the following: bilateral conjunctivitis, lesions of the lips and oral cavity ('strawberry tongue,' dry red chapped lips), peripheral
extremity erythema or edema (erythema of the palms and solesj indurative edema of the hands and feetj late desquamation
5 of the fingertips), polymorphous rash (primarily truncal), and/or cervical lymphadenopathy (often painfu l and unilateral, with
6 at least one node >1. 5 em) . Coronary aneurysms are a major complication and occur in up to 25% of untreated patients with
7 Kawasaki's disease. Intravenous immunog lobulin greatly reduces/prevents the likelihood of coronary aneurysm development.
High-dose aspirin is used for combating inflammation and fever. Other manifestations include sterile pyu ria, gallbladder
8
hyd rops, hepatitis, and arthrit is.
9 Kawasaki disease Pyuria Immunoglobulin therapy Coronary artery aneurysm Erythema DesQuamation Conjunctivitis Lymphadenopathy Aspirin Edema Gallbladder Hepatitis
10 Inflammation Intravenous therapy Antibody Mouth Cervical lymphadenopathy Arthritis Aneurysm Sole (foot) Rash Fever
11
A is not correct. 4 % chose this.
• 12
Aortic stenosis is not seen in patients with Kawasaki's disease. In children, aortic valvular issues are most commonly due to
• 13 bicuspid aortic valve.
• 14 Kawasaki disease Aortic stenosis Aortic valve Bicuspid aortic valve Stenosis

• 15
C is not correct. 8 % chose this .
• 16
Mitral va lve prolapse is not seen in children w ith Kawasaki's diseasej however, it can be a complication of rheumatic fever.
• 17 Kawasaki disease Mitral valve prolapse Rheumatic fever Mitral valve Fever Valve

• 18
D is not correct. 8 % chose this.
• 19
Myocardial infarction (MI) is actually a consequence of developing a coronary aneurysm . Patients with Kawasaki's disease are
• 20 at risk for the development of coronary aneu rysm because of the underlying associated vascu litis, and the MI is only a further
• 21 complication because of the presence of the aneurysm itself.
Kawasaki disease Coronary artery aneurysm Myocardial infarction Vasculitis Aneurysm Infarction
• 22
• 23 E is not correct. 1 0/o chose th is .
• 24

Lock Suspend
s 0
End Block
Item: 11 of 24 ~ 1 • Mark <:] (:>- Jill ~· ~J
QID: 23397 ..1 Previous Next Lab'V!I!ues Notes Calculator
• ••
1

2
Aortic stenosis is not seen in patients with Kawasaki 's disease. In ch ildren, aortic va lvu lar issues are most commonly due to
bicuspid aortic valve.
3 Kawasaki disease Aortic stenosis Aortic valve Bicuspid aortic valve Stenosis
4
C is not correct. 80/o chose this.
5
Mitral va lve prolapse is not seen in ch ildren w ith Kawasaki's disease; however, it can be a complication of rheumatic fever.
6 Kawasaki disease Mitral valve prolapse Rheumatic fever Mitral valve Fever Valve
7
D is not correct. 80/o chose this.
8
Myocardial infarction (MI) is actually a consequence of developing a coronary aneurysm . Patients with Kawasaki's disease are
9
at risk for the development of coronary aneurysm because of the underlying associated vascu litis, and the MI is only a further
10 complication because of the presence of the aneurysm itself.
11 Kawasaki disease Coronary artery aneurysm Myocardial infarction Vasculitis Aneurysm Infarction

• 12 E is not correct. 10/o chose this .


• 13 Pulmonary hypertension can be seen in patients with valvular heart problems, not in those with Kawasaki's disease.
• 14 Kawasaki disease Pulmonary hypertension Kawasaki Heavy Industries Motorcycle & Engine Hypertension

• 15
• 16
Bottom line:
• 17
Coronary aneurysms are a major complication of, and occur in up to 25% of, untreated patients with Kawasaki's disease .
• 18
Intravenous immunoglobulin can greatly reduce the risk of developing coronary aneurysms.
• 19 Kawasaki disease Immunoglobulin therapy Antibody Aneurysm

• 20
• 21
• 22 References:
FA Step 2 CK 9th ed pp 392-394
• 23
FA Step 2 CK 8th ed p 374
• 24

Lock
s
Suspend
0
End Block
Item: 11 of 24 ~ 1 • Mark <:] (:>- Jill ~· ~J
QID: 23397 ..1 Previous Next Lab'V!I!ues Notes Calculator
• ••
1

2
Aortic stenosis is not seen in patients with Kawasaki 's disease. In ch ildren, aortic va lvu lar issues are most commonly due to
bicuspid aortic valve.
3 Kawasaki disease Aortic stenosis Aortic valve Bicuspid aortic valve Stenosis
4
C is not correct. 80/o chose this.
5
Mitral va lve prolapse is not seen in ch ildren w ith Kawasaki's disease; however, it can be a complication of rheumatic fever.
6 Kawasaki disease Mitral valve prolapse Rheumatic fever Mitral valve Fever Valve
7
D is not correct. 80/o chose this.
8
Myocardial infarction (MI) is actually a consequence of developing a coronary aneurysm . Patients with Kawasaki's disease are
9
at risk for the development of coronary aneurysm because of the underlying associated vascu litis, and the MI is only a further
10 complication because of the presence of the aneurysm itself.
11 Kawasaki disease Coronary artery aneurysm Myocardial infarction Vasculitis Aneurysm Infarction

• 12 E is not correct. 10/o chose this .


• 13 Pulmonary hypertension can be seen in patients with valvular heart problems, not in those with Kawasaki's disease.
• 14 Kawasaki disease Pulmonary hypertension Kawasaki Heavy Industries Motorcycle & Engine Hypertension

• 15
• 16
Bottom line:
• 17
Coronary aneurysms are a major complication of, and occur in up to 25% of, untreated patients with Kawasaki's disease .
• 18
Intravenous immunoglobulin can greatly reduce the risk of developing coronary aneurysms.
• 19 Kawasaki disease Immunoglobulin therapy Antibody Aneurysm

• 20
• 21
• 22 References:
FA Step 2 CK 9th ed pp 392-394
• 23
FA Step 2 CK 8th ed p 374
• 24

Lock
s
Suspend
0
End Block
Item: 12 of 24 ~ 1 • Mark <:] (:>- Jill ~· ~J
QID: 23717 ..1 Previous Next Lab'V!I!ues Notes Calculator

1 •
A 20-year-old woman at 16 weeks' gestation who recently immigrated from I nd ia presents to the outpatient interna l
2 medicine clin ic with a ch ief complaint of an erythematous maculopapular rash that began on her face and spread to her
3 trunk and extremities. She also reports some diffuse bilateral joint pain . On review of systems, she reports recent fever,
4 coryza, and cough . The physician sends rubel la titers that come back posit ive for rubella IgM . She chooses not to terminate the
pregnancy and continues to receive regu lar prenatal care. She delivers vagina lly at 38 weeks without any complications .
5
Newborn is noted to have a continuous murmur at the left sternal border, but is acyanotic. 3 days after delivery the murmur
6 persists and the doctor recommends medical management of the infant's condition .
7
8 What is the mechan ism of action of the first- li ne agent?
9
10 A. Decrease in arterial oxygen saturation
11
B. Increase in arterial oxygen saturation
• 12
• 13 C. Increased production of prostaglandins
• 14 D. Inhibition of prostaglandin synthesis
• 15
E. Prevention of endocarditis
• 16
• 17
• 18
• 19
• 20
• 21
• 22
• 23
• 24

Lock
s
Suspend
0
End Block
Item: 12 of 24 ~ 1 • Mark <:] (:>- Jill ~· ~J
QID: 23717 ..1 Previous Next Lab'V!I!ues Notes Calculator

1
The correct answer is D. 800/o chose this.
2
Maternal first-trimester rubella infection increases a newborn's risk of sensorineural deafness, cataracts, cardiac defects such
3 as patent ductus arteriosus (PDA), behavioral disorders, and mental retardation . The baby's development of a continuous
4 murmur is indicative of the presence of a PDA, wh ich is kept open in utero by low arterial oxygen saturation and
5
prostagland ins released by the placenta . Indomethacin, a prostaglandin synthesis inhibitor, is first-line treatment for closure
of PDAs .
6 Patent ductus arteriosus Prostaglandin Placenta lndometadn Rubella Ductus arteriosus Intellectual disability Cataract Oxygen saturation Oxygen Hearing loss Pregnancy Infection
7 Sensorineural hearing loss
8
A is not correct. 1% chose this.
9
Exposure to rubella during the first trimester and the presence of the continuous murmur indicate that the infant has a patent
10
ductus arteriosus ( PDA) . Decreased arterial oxygen saturation maintains the PDA in utero instead of contributing to PDA
11 closu re. After delivery, the onset of respiration results in increased oxygen saturation . Increased saturation resu lts in
12 constriction of the ductus arteriosus, contributing to its closure.
Patent ductus arteriosus Ductus arteriosus Oxygen saturation Oxygen saturation (medicine) Oxygen Respiration (physiology) Rubella In utero Pregnancy
• 13
• 14 B is not correct. 30/o chose this .
• 15 An increase in arterial oxygen saturation results from the in itiation of respiration after birth . Increased oxygen saturation
• 16 contributes to closure of a patent ductus arteriosus ( PDA) , but is not a treatment option for medical management of PDAs .
Patent ductus arteriosus Oxygen saturation (medicine) Ductus arteriosus Oxygen saturation Oxygen Respiration (physiology) Personal digital assistant
• 17
• 18 C is not correct. 160/o chose this .
• 19 Prostaglandins maintain patency of a patent ductus arteriosus (PDA) instead of contributing to its closure. Prostaglandins are
believed to be released by the placenta and contribute to maintaining the PDA in utero . This is believed to occur through the
• 20
cyclooxygenase-2 pathway. Prostag land in E1 is given after birth to maintain PDAs in patients with ductus-dependent lesions
• 21 (transposition of the great vessels, coarctation of the aorta, or obstruction of the pu lmonic valve) .
• 22 Transposition of the great vessels Patent ductus arteriosus Coarctation of the aorta Pulmonary valve Placenta Prostaglandin Aorta Ductus arteriosus In utero

• 23 Prostaglandin-endoperoxide synthase 2 Great vessels

• 24

Lock
s
Suspend
0
End Block
Item: 12 of 24 ~ 1 • Mark <:] (:>- Jill ~· ~J
QID: 23717 ..1 Previous Next Lab'V!I!ues Notes Calculator

1 B is not correct. 30/o chose this .


2 An increase in arterial oxygen saturation results from the in itiation of respiration after birth . Increased oxygen saturation
3 contributes to closure of a patent ductus arteriosus (PDA), but is not a treatment option for medical management of PDAs .
Patent ductus arteriosus Oxygen saturation (medicine) Ductus arteriosus Oxygen saturation Oxygen Respiration (physiology) Personal digital assistant
4

5 C is not correct. 160/o chose this .


6 Prostaglandins mainta in patency of a patent ductus arteriosus {PDA) instead of contributing to its closure. Prostaglandins are
7
believed to be released by the placenta and contribute to maintain ing the PDA in utero . This is believed to occur through the
cyclooxygenase-2 pathway. Prostagland in E1 is given after birth to maintain PDAs in patients with ductus-dependent lesions
8
(transposition of the great vessels, coarctation of the aorta, or obstruction of the pulmonic valve) .
9 Transposition of the great vessels Patent ductus arteriosus Coarctation of the aorta Pulmonary valve Placenta Prostaglandin Aorta Ductus arteriosus In utero

10 Prostaglandin-endoperoxide synthase 2 Great vessels

11
E is not correct. OO/o c hose th is.
12
Patients with a patent ductus arteriosus are not necessarily at higher risk for endocardit is. Endocarditis is associated with
• 13 congenital valvular defects and coarctation of the aorta, and can be medically managed with antibiotics prior to surgeries or
• 14 dental procedures .
Patent ductus arteriosus Coarctation of the aorta Endocarditis Aorta Ductus arteriosus Congenital disorder Antibiotics
• 15
• 16
• 17 Bottom line :
• 18
Indomethacin, a prostaglandin synthesis inhibitor, is first-li ne treatment for closure of patent ductus arteriosus .
• 19 Patent ductus arteriosus Indometacin Prostaglandin Ductus arteriosus Patent Enzyme inhibitor

• 20
• 21
• 22 References:
FA Step 2 CK 9th ed pp 373-375
• 23
FA Step 2 CK 8th ed pp 357-358
• 24

Lock
s
Suspend
0
End Block
Item: 13 of 24 ~ 1 • Mark <:] (:>- Jill ~· ~J
QID: 21543 ..1 Previous Next Lab'V!I!ues Notes Calculator

1 •
A 7-yea r- old girl presents to the emergency department complaining of chest pa in and pain in her lower extremities that
2 occurred twice earli er that day while she was running at school. The chest pain is described as left-sided, dull,
3 nonradiating, and accompanied by a headache. These symptoms have occurred previously, although the patient cannot
4 recall when these episodes first began . Her temperatu re is 37°C (98 .6°F), pulse is 140/min, blood pressure is 135/90 mm Hg,
and respiratory rate is 22/min. Her lungs are clear, and cardiac examination reveals normal 5 1 and 5 2 sounds, with a systolic
5
ej ection murmur at the left upper sternal border radiating to the interscapu lar region, best heard over her back between her
6 scapulae. X-ray of her chest is shown in the image.
7
8
9
10
11
12
• 13
• 14
• 15
• 16
• 17
• 18
• 19
• 20
Image courtesy of Koletsis, eta!. Figure 1: J Cardiothorac Surg. 2009; 4:10
• 21
• 22
What additional findings would you expect in this patient?
• 23
• 24

Lock
s
Suspend
0
End Block
Image courtesy of Koletsis, eta/. Figure 1: J Cardiothorac Surg. 2009; 4:10
• 13
• 14
What additional findings would you expect in this patient?
• 15
• 16
A. A sudden episode of cyanosis that is partially relieved by squatting
• 17
• 18 B . Blue skin and general fa il ure to thrive
• 19 C. Decreased breath sounds over bilateral lung bases
• 20
D . Norma l additional physical exam fi ndings
• 21
• 22 E. Notching of the r ibs
• 23

• 24

a
Lock
s
Suspend
8
End Block
Item: 13 of 24 ~ 1 • Mark <:] (:>- Jill ~· ~J
QID: 21543 ..1 Previous Next Lab'V!I!ues Notes Calculator

1 The correct answer is E. 510/o chose this.


2 The patient has coarctation of the aorta, a cause of acyanotic heart disease. Coarctation of the aorta is a narrowing of the
3 aorta, typically of the proximal thoracic aorta immediately distal to the left subclavian artery. Coarctation of the aorta
4 accounts for 8% of congenita l heart defects, w ith a 2:1 male:female predominance. It may be associated with a bicuspid
aortic valve in 80% of cases . In approximately 10% of cases, congestive heart failure may develop in infancy. Physica l
5
examination is rema rkable for a systolic ejection murmur at the left upper sternal border radiating to the interscapular region .
6 Although not mentioned in this case, our patient wou ld most likely have weak and delayed femoral pu lses relative to the
7 upper extrem ity pulses, and hypertension in the upper extremities. Coarctation of the aorta can lead to collateral circulation
8
through the intercostal arteries and erosion of the ribs by col laterals, resulting in notch ing of the ribs, as seen on this
patient's chest x- ray.
9 Coarctation of the aorta Subclavian artery Bicuspid aortic valve Heart failure Intercostal arteries Aorta Aortic valve Congestive heart failure Thoracic aorta Notching of the ribs
10 Hypertension Systole Artery Congenital heart defect Congenital disorder Anatomical terms of location Mitral valve Cardiovascular disease Thorax Thoracic vertebrae Sternum
11 Heart murmur Physical examination Femoral artery Stenosis
12
13
• 14

• 15
• 16
• 17
• 18
• 19
• 20
• 21 Image courtesy of Koletsis, et
at. Figure 1: J Cardiothorac
• 22
Surg. 2009; 4:10
• 23
• 24 A is not correct. 2 6 0/o chose this .

Lock
s
Suspend
0
End Block
Item: 13 of 24 ~ 1 • Mark <:] (:>- Jill ~· ~J
QID: 21543 ..1 Previous Next Lab'V!I!ues Notes Calculator

1 • Surg. 2009; 4:10


2 A is not correct. 260/o chose this.
3 This finding is associated with tetralogy of Fallot (TOF) . TOF is composed of four defects : right ventricu lar outflow tract
4 obstruction, ventricular septal defect, right ventricu lar hypertrophy, and an overriding aorta . TOF usually presents in infancy
5 w ith cyanotic spells, a normal S1, single S2, and a harsh systol ic crescendo-decrescendo ejection murmur that is best heard
over the left upper sternal border, with radiation to the back. The cyanotic spells of TOF often can be relieved by squatting,
6
which increases systemic vascular resistance and reverses the shunting of blood flow.
7 Tetralogy of Fallot Ventricular septal defect Overriding aorta Aorta Right ventricular hypertrophy Cyanotic heart defect Cyanosis Ventricular hypertrophy Systole Hypertrophy
8 Vascular resistance Ventricular outflow tract Blood vessel Ventricular outflow tract obstruction Sternum Heart murmur
9
B is not correct. ]Ofo chose this.
10
This finding is associated with dextroposed transposition of the great arteries ( D-TGA), in which the aorta arises from the
11 right ventricle and the pulmonary artery from the left ventricle. D-TGA usually presents at birth w ith cyanosis and tachypnea .
12 The missing main pulmonary artery stem and sma ll heart base produce an egg-shaped silhouette on x-ray of the chest.
13 Transposition of the great arteries is a severe cyanotic heart disease and generally not compatible with life beyond infancy.
Cyanosis Tachypnea Transposition of the great vessels Cyanotic heart defect Aorta Ventricle (heart) Pulmonary artery Artery Great arteries Cardiovascular disease X-ray
• 14
• 15 C is not correct. 6% chose this.
• 16 Bilateral decreased breath sounds at the lung bases and blunted costophren ic ang les usually are associated with pleural
effusions. The patient's presentation and physical findings make th is diagnosis un likely.
• 17
lung Costodiaphragmatic recess Pleural cavity Respiratory sounds
• 18
• 19 D is not correct. 100/o chose this .
• 20 Given this patient's age, chest x-ray and the presence of a systolic murmur indicating significant collatera l formation, a
completely norma l physical exam is less likely. She w ill likely have cool lower extremities w ith decreased pulses as compared
• 21 to her upper extremities before she develops symptoms .
• 22 Chest radiograph Systole X-ray Heart murmur Physical examination

• 23
• 24

Lock
s
Suspend
0
End Block
Item: 13 of 24 ~ 1 • Mark <:] (:>- Jill ~· ~J
QID: 21543 ..1 Previous Next Lab'V!I!ues Notes Calculator

• • • • • •
right ventricle and the pulmonary artery from the left ventricle. D-TGA usually presents at birth with cyanosis and tachypnea .
2
The m issing ma in pulmonary artery stem and small heart base produce an egg-shaped silhouette on x-ray of the chest.
3 Transposition of the great arteries is a severe cyanotic heart disease and generally not compatible with life beyond infancy.
4 Cyanosis Tachypnea Transposition of the great vessels Cyanotic heart defect Aorta Ventricle (heart) Pulmonary artery Artery Great arteries Cardiovascular disease X-ray

5 C is not correct. 6 0/o chos e this .


6 Bilateral decreased breath sounds at the lung bases and blunted costoph renic angles usually are associated with pleural
7 effusions. The patient's presentation and physical findings make this diagnosis unlikely.
lung Costodiaphragmatic recess Pleural cavity Respiratory sounds
8
9 D is not correct. 100/o chose this .
10 Given this patient's age, chest x-ray and the presence of a systolic murmur indicating significant collateral formation, a
11 completely normal physical exam is less likely. She w ill likely have cool lower extremities w ith decreased pulses as compared
to her upper extremities before she develops symptoms .
12
Chest radiograph Systole X-ray Heart murmur Physical examination
13
• 14

• 15 Bottom line :
• 16 Coarctation of the aorta, an acyanotic heart disease, can lead to collateral ci rculation through the intercostal arteries and
• 17
erosion of the ribs by collaterals, resulting in rib notchirng . Physical exam is remarkable for a systolic ejection mu rmur at the
left upper sternal border radiating to the interscapular region, weak and delayed femoral pulses relative to the upper
• 18
extremity pu lses, and hypertension in the upper extremities.
• 19 Coarctation of the aorta Intercostal arteries Aorta Hypertension Artery Cardiovascular disease Heart murmur Femur Sternum Systole Rib

• 20
• 21
• 22 References:
FA Step 2 CK 9th ed pp 371-378
• 23
FA Step 2 CK 8th ed pp 355-360
• 24

Lock
s
Suspend
0
End Block
Item: 14 of 24 ~ 1 • Mark <:] (:>- Jill ~· ~J
QID: 21560 ..1 Previous Next Lab'V!I!ues Notes Calculator

1 •
A 4-year-old boy is brought to the emergency department by his parents after they noticed he was agitated, breathing ~~AI
2 hard, and appeared bluish . This happens occasionally, but generally he will assume a squatting position and feel better. His
3 parents have never seen him this agitated and he appears more cyanotic than normal. Past medical history is notable for a
4 spontaneous vaginal delivery at 33 weeks' gestation and congen ita l heart disease diagnosed during infancy that causes episodes
of cyanosis. He is afebrile with a heart rate 120/min, blood pressure of 99/58 mm Hg, respiratory rate of 30/min, and oxygen
5
saturation is 80% on room air. He is restless, cyanotic, and dyspneic. He has a right ventricular impu lse; a norma l Sl; a loud
6 single 52; and a harsh systolic crescendo-decrescendo ejection murmur heard best over the left upper sterna l border, radiating
7 to the back . His lungs are clear. He has clubbing of the digits of his upper extremities.
8
9 Which of the following is the most appropriate next step in management"'
10
11 A. Administer digoxin
12
B. Administer heparin
13
• 14
C. Administer phenobarbital

• 15 D. Cardiac catheterization
• 16
E. Emergent surgery to correct cardiac defect
• 17
F. Placement of the patient in a knee-chest posit ion
• 18
• 19
• 20
• 21
• 22
• 23
• 24

Lock
s
Suspend
0
End Block
Item: 14 of 24 ~ 1 • Mark <:] (:>- Jill ~· ~J
QID: 21560 ..1 Previous Next Lab'V!I!ues Notes Calculator

1
The correct answer i s F. 610/o chose th is.
2
This patient has tetralogy of Fallot (TOF), which consists of four defects: right ventricu lar outflow tract obstruction, ventricular
3 septal defect, rig ht ventricular hypertrophy, and an overriding aorta . TOF usually presents in infancy, with cyanotic spells, a
4 normal 51, single 52, and a harsh systolic crescendo-decrescendo ejection murmur that is best heard over the left upper
5
sternal border and radiates to the back. Patients with severe TOF, as in the case above, will experience episodes of
hypoxemia, or "tet spells," resulting from reduction in pulmonary blood flow. These spells are characterized by ( 1) cyanosis,
6
(2) sudden-onset dyspnea, (3) alterations in consciousness/agitation, and ( 4) a decrease in the intensity of the murmur due
7 to decreased flow across the obstruction . Management of these spells beg ins w ith placing the ch ild in t he knee-chest posit ion
8 to increase systemic vascular resistance. Further steps in management include administering supplemental oxygen to
increase the delivery of oxygen, and morphine su lfate for sedation and to relax the pulmonary arteries.
9
Tetralogy of Fallot Ventricular septal defect Cyanosis Overriding aorta Dyspnea Hypoxemia ~1orphine Aorta Right ventricular hypertrophy Cyanotic heart defect Systole
10
Pulmonary artery Ventricular hypertrophy Hypertrophy Artery Vascular resistance Ventricular outflow tract Sternum Sedation Heart murmur Oxygen Blood vessel Ventricle (heart)
11
12 A i s not correct. 3% chose this.
13
Digoxin and diuretics sometimes are considered in patients with tetralogy of Fallot and increased pulmonary blood flow that
requ ires treatment for heart fa ilure. These medications are discontinued when the right ventricular tract obstruction worsens.
14 Tetralogy of Fallot Digoxin Diuretic Heart failure
• 15
B is not correct. 2 0/o chose this .
• 16
Heparin is used when immediate anticoagulation is needed . I t can be used in the setting of pulmonary embolism, acute
• 17
coronary syndrome, deep venous thrombosis, and cerebrovascular accident. Given the history and physical exam findings,
• 18 there is no ind ication to use hepa rin so it would not be the next best step in management.
• 19 Pulmonary embolism Deep vein thrombosis Acute coronary syndrome Stroke Heparin Anticoagulant Venous thrombosis Thrombosis Embolism Physical examination

• 20 Cerebrovascular disease Vein

• 21 C i s not correct. 3 0/o chose this .


• 22 Phenobarbital is administered for seizures and is not the most appropriate next step. Given the patient's physical exam
• 23 findings, the etiology of his symptoms is not neurologic in nature so phenobarbita l is not indicated in this situation .
Phenobarbital Epileptic seizure Etiology Neurology
• 24

Lock
s
Suspend
0
End Block
Item: 14 of 24 ~ 1 • Mark <:] (:>- Jill ~· ~J
QID: 21560 ..1 Previous Next Lab'V!I!ues Notes Calculator
~, I • T • II • • • t T• I II • I • I • 'I' T I • t•• -•t. • I.IJ. T• II II I II II "T • ••. • I
1
Cerebrovascular disease Vein
2
3 C is not correct. 3% chose this.
4 Phenobarbital is administered for seizures and is not the most appropriate next step . Given the patient's physical exam
5
findings, the etiology of his symptoms is not neurologic in nature so phenobarbita l is not ind icated in this situation .
Phenobarbital Epileptic seizure Etiology Neurology
6
7 D is not correct. 6 % chose this.
8 Cardiac catheterization can be considered; however, acutely the patient needs supplemental oxygen as well as other
interventions . It is important to stabilize the patient as much as possible before considering further diagnostic studies.
9 Cardiac catheterization Catheter Oxygen Urinary catheterization Oxygen mask
10
E is not correct. 2 5 % chose this.
11
The patient is having a "tet spell" that can be managed medically. Surgery should be considered to repair his defect; however,
12
emergent surgery is not necessary. Emergent surgery can be considered in patients with medically refractory tet spells or for
13 those with severe right ventricular outflow tract obstruction not amenable to medical management.
14 Ventricular outflow tract Ventricular outflow tract obstruction

• 15
• 16
Bottom line :
• 17
Management of "tet spells" includes placing the child in the knee-chest position, administering supplemental oxygen, and
• 18
morphine sulfate.
• 19 Morphine Oxygen Sulfate

• 20
• 21
• 22 References:
FA Step 2 CK 9th ed pp 371-378
• 23
FA Step 2 CK 8th ed pp 355-360
• 24

Lock
s
Suspend
0
End Block
Item: 15 of 24 ~ 1 • Mark <:] (:>- Jill ~· ~J
QID: 23642 ..1 Previous Next Lab'V!I!ues Notes Calculator

1
2

Four hours after birth, a neonate w ithout dysmorph ic features has noticeable periorbital and perioral cyanosis, markedly so 1•AI
when crying . The pregnancy was uncompl icated, the infant was born at term, and her mother was negative for Group B A
3 Streptococcus. Vital signs are stable, but the infant's oxygen saturation is 86% on room air. The lungs are clear to
4 auscultation, but a continuous, mach ineli ke murmur is heard along the left sternal border. Arterial partial pressure of oxygen
increases to 110 mm Hg on 100% oxygen for 10 minutes . An echocardiogram is obtained and is shown in the image .
5
6
7
8
9
10
11
12
13
14
• 15
• 16
• 17
• 18
• 19
• 20
• 21
• 22
• 23
• 24

Lock
s
Suspend
0
End Block
12
13 Which of the following is the best next step in management of this patient?
14
• 15
A . Administer furosemide
• 16
B . Administer prostag landin E1
• 17
• 18 C. Indomethacin
• 19
D. Obtain a cardiac catheterization
• 20
E. Obtain an x-ray of the chest
• 21
• 22 F. Schedu le the patient for emergent corrective surgery
• 23

• 24

a
Lock
s
Suspend
8
End Block
Item: 15 of 24 ~ 1 • Mark <:] (:>- Jill ~· ~J
QID: 23642 ..1 Previous Next Lab'V!I!ues Notes Calculator

2 The correct answer is B. 540/o chose th is.


3 The echocardiogram clearly displays a pulmonary artery arising from the left ventricle, part of transposition of the great
4
arteries . The other component of the condition is that the aorta is arising from the right ventricle (not seen in this view) .
Systemic and pulmonary circulations exist in parallel, with oxygenated blood returning to the left ventricle but being pumped
5 back into the pulmonary circulation . This causes the patient's central cyanosis, and it is not worse on ly because a patent
6 ductus arteriosus {PDA), identified by the characteristic murmur, allows m ixing of pulmonary and systemic circulation .
7 Prostaglandin E1 keeps the PDA patent to allow oxygenated blood into the periphery until the inevitable surgery.
Cyanosis Patent ductus arteriosus Echocardiography Prostaglandin Pulmonary artery Transposition of the great vessels Pulmonary circulation Aorta Ventricle (heart) Circulatory system
8
Ductus arteriosus Artery Blood Great arteries Ventricular system Heart murmur
9
10 A is not correct. 3% chose this.
11 Furosemide is usefu l for managing symptoms of heart fa'ilure in an infant. This patient could develop such symptoms if
untreated, presuming she has a ventricular septal defect, a common abnormality occurring w ith transposit ion of the great
12
arteries . However, this does not prevent the infant's systemic hypoxemia from worsen ing .
13 Furosemide Ventricular septal defect Transposition of the oreat vessels Hypoxemia Heart failure Artery Great arteries
14
C is not correct. 2 8 0/o chose this.
15
Indomethacin wou ld close the patent ductus arteriosus . It would be catastrophic to give indomethacin to a newborn with
• 16
transposition of the great arteries as the only thing that is keeping the blood oxygenated and circulating through the baby is
• 17 the patent ductus arteriosus .
• 18 Patent ductus arteriosus Indometacin Transposition of the great vessels Ductus arteriosus Artery Great arteries

• 19 D is not correct. 3% chose this.


• 20 A cardiac catheter may be useful in diagnosing transposit ion of the great arteries when the echocardiogram is not diagnostic
• 21 (but it is here) . It can also be used for a balloon atrial septostomy when a patent ductus arteriosus {PDA) is not present, but
• 22
because this patient has a PDA, she needs prostaglandin to maintain it open to allow for mixing of systemic and pulmonary
blood flow.
• 23 Patent ductus arteriosus Echocardiography Prostaglandin Transposition of the great vessels Cardiac catheterization Catheter Ductus arteriosus Artery Great arteries
• 24

Lock
s
Suspend
0
End Block
Item: 15 of 24 ~ 1 • Mark <:] (:>- Jill ~· ~J
QID: 23642 ..1 Previous Next Lab'V!I!ues Notes Calculator

1
D i s not correct. 3% chose this.
2 A cardiac catheter may be useful in diagnosing transposit ion of the great arteries when the echoca rdiogram is not diagnostic
3 (but it is here) . It can also be used for a balloon atrial se ptostomy when a patent ductus arteriosus (PDA) is not present, but
4 because this patient has a PDA, she needs prostaglandin to maintain it open to allow for mixing of systemic and pulmonary
blood flow.
5
Patent ductus arteriosus Echocardiography Prostaglandin Transposition of the great vessels Cardiac catheterization Catheter Ductus arteriosus Artery Great arteries
6
E i s not corr ect. 30/o c hose t his.
7
An x-ray of the chest wou ld not be helpful at this point because the echocardiogram is diagnostic, but the plain film may show
8
an egg-shaped silhouette (owing to a main pulmonary artery segment not appearing as the characteristic notch on the fil m) .
9 The hyperoxia test ( 10 m inutes of 100% oxygen) did not produce an arterial partial pressure of oxygen > 150 mm Hg,
10 indicating cardiac and not pulmonary pathology.
Partial pressure Echocardiography Hyperoxia Pulmonary artery X-ray Radiography Oxygen Pathology
11
12 F is not correct. 9 0/o chose this.
13 Although the patient w ith transposition of the great arter ies does demand corrective surgery, it is much more important to
14 administer prostaglandin to the patient with a patent ductus arteriosus (PDA) to ensure oxygenated blood can mix with the
systemic circulation and prevent worsen ing cyanosis . Without an atrial septal defect, ventricular septal defect or PDA
15
transposition is incompatible with life .
• 16 Atrial septal defect Ventricular septal defect Cyanosis Patent ductus arteriosus Transposition of the great vessels Prostaglandin Ductus arteriosus Circulatory system Great arteries
• 17 Artery Blood Atrium (heart) Ventricle (heart)
• 18
• 19
Bottom Line :
• 20
• 21 In transposition of the great arteries the pu lmonary artery arises from the left ventricle and the aorta arises from the right
ventricle. A patent ductus arteriosus allows mixing of pulmonary and system ic circulation, and should be kept open with
• 22
prostaglandin E1 to allow oxygenated blood into the periphery until surgical repair can be performed .
• 23 Patent ductus arteriosus Pulmonary artery Prostaglandin Transposition of the great vessels Aorta Ventricle (heart) Ductus arteriosus Circulatory system Great arteries Artery Blood

• 24 Ventricular svstem

Lock
s
Suspend
0
End Block
Item: 15 of 24 ~ 1 • Mark <:] (:>- Jill ~· ~J
QID: 23642 ..1 Previous Next Lab'V!I!ues Notes Calculator
. .. .. . .. . .. . . .. . . .. .. . . . .. . .. .. .. .. .. . - . . . . .. . .. . .. . .. .. - .. . .
1

2 E i s not corr ect. 30/o c hose th is.


3 An x-ray of the chest wou ld not be helpful at this point because the echocardiogram is diagnostic, but the plain film may show
an egg-shaped silhouette (owing to a main pulmonary artery segment not appearing as the characteristic notch on the fil m) .
4
The hyperoxia test (10 m inutes of 100% oxygen) did not produce an arterial partial pressure of oxygen >1 50 mm Hg,
5 indicating card iac and not pulmonary pathology.
6 Partial pressure Echocardiography Hyperoxia Pulmonary artery X-ray Radiography Oxygen Pathology

7 F is not correct. 9 0/o chose this.


8 Although the patient with transposit ion of the great arter ies does demand corrective su rgery, it is much more important to
9 admin ister prostagland in to the patient with a patent ductus arteriosus ( PDA) to ensure oxygenated blood can mix with the
10 system ic circulation and prevent worsening cyanosis. Without an atrial septal defect, ventricular septal defect or PDA
t ransposit ion is incompatible with life.
11 Atrial septal defect Ventricular septal defect Cyanosis Patent ductus arteriosus Transposition of the great vessels Prostaglandin Ductus arteriosus Circulatory system Great arteries
12
Artery Blood Atrium (heart) Ventricle (heart)
13
14

15 Bottom line :
• 16 In transposition of the great arteries the pu lmonary artery arises from the left ventricle and the aorta arises from the right
• 17
ventricle. A patent ductus arteriosus allows mixing of pulmonary and system ic circulation, and should be kept open with
prostaglandin E1 to allow oxygenated blood into the periphery until surgical repair can be performed .
• 18 Patent ductus arteriosus Pulmonary artery Prostaglandin Transposition of the great vessels Aorta Ventricle (heart) Ductus arteriosus Circulatory system Great arteries Artery Blood
• 19 Ventricular system

• 20
• 21
• 22 Refer en ces :
FA Step 2 CK 9th ed pp 376- 377; 378
• 23
FA Step 2 CK 8th ed p 359
• 24

Lock
s
Suspend
0
End Block
Item: 16 of 24 ~ 1 • Mark <:] (:>- Jill ~· ~J
QID: 23645 ..1 Previous Next Lab'V!I!ues Notes Calculator

1 •
A 16- month-old boy is brought to his ped iatrician because his parents have noticed that occasionally his li ps turn blue .
2 They mention that too much activity can bring on the change in color of his lips, and he has to stop what he is doing and
3 squat for a short period . Review of records reveals his par ents have m issed his previous appointments on multiple
4 occasions. On cardiac auscultation, the patient has a single second heart sound and a systolic ejection crescendo-decrescendo
murmur heard best along the left upper sternal border. An echocardiogram reveals the origination of the aortic root to be
5
deviated to the right.
6
7
Which other abnormality is most likely to be seen on further echocardiography?
8
9
A. Atrial septal defect (either ostium primum or ostium secundum)
10
11
B. Left ventricular dilation
12 C. Left ventricular outflow tract obstruction
13
D. Patent ductus arteriosus
14
15
E. Right ventricular dilation
• 16 F. Ventricular septal defect
• 17
• 18
• 19
• 20
• 21
• 22
• 23
• 24

Lock
s
Suspend
0
End Block
Item: 16 of 24 ~ 1 • Mark <:] (:>- Jill ~· ~J
QID: 23645 ..1 Previous Next Lab'V!I!ues Notes Calculator

2 The correct answer is F. 650/o chose this.


3 This patient has tetralogy of Fallot (TOF), consisting of right ventricula r outflow tract obstruction (pulmonary stenosis or
4
atresia), at least one ventricular septal defect (VSD), a right-deviated aortic root overriding t he VSD, and concentric right
ventricula r (RV) hypertrophy. This is the most common cyanotic congenita l heart condition . It can present immediately at
5 birth or progressively as it does in this case. The RV outflow t ract obstruction accounts for the characteristic mu rmur, and the
6 single 5 2 is due to an inaudible pu lmonic component. The classic "tet spells" described are believed to be due to pulmonary
7 infundibulum spasms and are alleviated by squatting, which increases systemic vascula r resistance . This, in tu rn, decreases
shunting across the VSD (since pulmonary resistance is less than systemic vascu lar resistance), thereby allowing more blood
8
to enter pulmonary circulation for oxygenation . The VSD in TOF is most commonly a single large mala ligned subaortic defect
9 located in the perimembranous region of the septum . The VSD can extend into the muscular septum, but there are rarely
10 other muscular VSDs.
Tetralogy of Fallot Ventricular septal defect Pulmonary circulation Pulmonic stenosis Cyanosis Cyanotic heart defect Stenosis Aorta Infundibulum {heart) Hypertrophy
11
Congenital disorder Septum Ventricular outflow tract Vascular resistance Atresia Ascending aorta Blood vessel Ventricle (heart) Oxygen saturation (medicine) Heart murmur
12
13 A is not correct. 4% chose this.
14 Although a chi ld with tetralogy of Fallot may have an atria l septal defect, it is not part of the tetralogy (ventricula r septal
15 defect, right deviated overriding aorta, pulmon ic stenosis, right ventricula r hypertrophy) that is found when diagnosing the
congenita l heart malformation .
16
Atrial septal defect Tetralogy of Fallot Overriding aorta Aorta Congenital heart defect Pulmonic stenosis Ventricular hypertrophy Stenosis Right ventricular hypertrophy
• 17
Congenital disorder Hypertrophy Atrium (heart) Pulmonary valve stenosis
• 18
• 19
B is not correct. 20/o chose this .
• 20
A child with tetralogy of Fallot, by defin ition, has concent ric right ventricular (RV) hypertrophy, but left ventricular dilation
would not be expected . The RV hypertrophy is thought to be a response to the large VSD and right ventricular outflow
• 21 obstruction with resultant systemic right ventricular systolic pressure .
• 22 Tetralogy of Fallot Hypertrophy Systole Cardiac cycle Heart Blood pressure Vasodilation Ventricle (heart)

• 23
C is not correct. 100/o chose this .
• 24

Lock
s
Suspend
0
End Block
Item: 16 of 24 ~ 1 • Mark <:] (:>- Jill ~· ~J
QID: 23645 ..1 Previous Next Lab'V!I!ues Notes Calculator

1 C is not correct. 100/o chose this.


2 A child with tetralogy of Fallot, by definition, has stenosis of the pulmonary artery leading to right ventricu lar outflow tract
3 obstruction . It would not be common to also find left ventricular outflow tract obstruction .
Tetralogy of Fallot Pulmonary artery Stenosis Stenosis of pulmonary artery Heart Ventricular outflow tract
4

5 D is not correct. 7 % chose this.


6 Although a ch ild with tetralogy of Fallot may have a paternt ductus arteriosus, it is not part of the tetralogy (ventricu lar septal
7
defect, right deviated overriding aorta, pu lmonic stenosis, right ventricular hypertrophy) that is found when diagnosing the
congenital heart malformation .
8 Tetralogy of Fallot Patent ductus arteriosus Overriding aorta Aorta Congenital heart defect Pulmonic stenosis Ventricular hypertrophy Stenosis Right ventricular hypertrophy
9 Ductus arteriosus Congenital disorder Hypertrophy Pulmonary valve stenosis
10
E is not correct. 1 20/o chose this.
11
A child with tetralogy of Fallot, by definition, has concentric right ventricular (RV) hypertrophy. At a very advanced stage, the
12
RV may fail and dilate, but at this point the expected find ing is hypertrophy. The RV hypertrophy is thought to be a response
13 to the large VSD and right ventricular outflow obstruction w ith resu ltant system ic right ventricular systolic pressure.
14 Tetralogy of Fallot Hypertrophy Systole Cardiac cycle Blood pressure Ventricular system Vasodilation

15
16
Bottom line :
• 17
Tetralogy of Fallot consists of right ventricular outflow tract obstruction, a ventral septal defect, a right-deviated aortic root
• 18
overriding the ventral septal defect, and concentric right ventricu lar hypertrophy.
• 19 Tetralogy of Fallot Right ventricular hypertrophy Ventricular hypertrophy Hypertroph y Aorta Ascending aorta

• 20
• 21
• 22 References:
FA Step 2 CK 9th ed pp 377; 378
• 23
FA Step 2 CK 8th ed pp 359-360
• 24

Lock
s
Suspend
0
End Block
Item: 17 of 24 ~ 1 • Mark <:] (:>- Jill ~· ~J
QID: 21554 ..1 Previous Next Lab'V!I!ues Notes Calculator

1 •
A 1-week-old infant presents to her general pediatrician 's office for a well-child visit. She was born at 39 weeks' gestation
2
to a GlPl mother without complications. Her temperature is 37 .0°C (98 .6°F), pu lse is 130/min, blood pressure is 72/54
3 mm Hg, and respiratory rate is 28/min . She is currently a.t the 50th percentile for weight and 75th percentile for height.
4 She is acyanotic and has a wide, fixed split 52, with a 3/6 systolic ejection murmur over the second intercostal space and is not
associated with a thrill. Sl is also split with a loud second component that is best heard over the left lower sterna l border. The
5
remainder of the exam ination is unremarkable.
6
7
Which of the following is the most likely diagnosis?
8
9
A. Atria l septal defect
10
11
B. Coarctation of the aorta
12 C. Dextratransposition of the great arteries
13
D. Tetralogy of Fallot
14
15
E. Ventricular septal defect
16
• 17
• 18
• 19
• 20
• 21
• 22
• 23
• 24

Lock
s
Suspend
0
End Block
Item: 17 of 24 ~ 1 • Mark <:] (:>- Jill ~· ~J
QID: 21554 ..1 Previous Next Lab'V!I!ues Notes Calculator

1
The correct answer is A. 640/o chose this.
2
Atrial septal defects {ASDs) are the second most common congenital heart lesion behind ventricular septum defect. ASDs can
3
be classified into two most common types : primum ASD results from arrested or abnorma l development of the endocardial
4 cushions, which contribute the apical portion of the atrial septum. This form of ASD is often associated with anterior m it ral
5 valve cleft and atrioventricular canal defects. Secundum ASD is caused by arrested growth of the secundum septum or
6
excessive absorption of the primum septum, resulting in a defect between the atria. This type of ASD is usually an isolated
finding . Patients with ASDs smaller than 7 to 8 mm in diameter often see a spontaneous closure of the defect, but in patients
7 w ith uncorrected moderate to large ASDs left-to-right shu nting may increase w ith age . The right-sided volume overload
8 associated w ith an ASD is usually well tolerated for years. Pulmonary vascular disease develops in about 10% of older
9 patients w ith isolated ASDs, but ra re in childhood and adolescence . ASDs are generally found incidentally during a routine
physical examination as they are small and do not cause symptoms in infancy and childhood . On physical exam, a wide, fixed
10
split 52 can be heard . The Sl heart sound may also be split with the second component (tricuspid closu re) intensified, wh ich
11 can be best heard at the lower left sternal border. A midsystolic pulmonary flow or ej ection murmur is classically present in
12 moderate to large left-to-right shunts due to the increased blood flow across the pulmonic valve. This murmu r is the loudest
13 over the second intercostal space . Untreated, patients may prog ress to congestive heart failure in 2nd and 3rd decades of life
and strokes due to paroxysmal emboli. Persistent left-to-right shunting w ith increased blood flow through the pulmonary
14
circulation can lead to pulmonary hypertension and right ventricular hypertrophy over t ime, eventually leading to a right-to-
15 left shunt called Eisenmenger syndrome.
16 Eisenmenger's syndrome Pulmonary valve Endocardial cushions Pulmonary hypertension Heart failure Pulmonary circulation Congestive heart failure Atrium {heart) Mitral valve

17 Right ventricular hypertrophy Interventricular septum Interatrial septum Heart sounds Hypertension Ventricular hypertrophy Split 52 Right-to-left shunt Tricuspid valve

• 18 Congenital disorder Endocardium Septum Embolism Hypertrophy Tricuspid atresia Sternum Physical examination Atrioventricular canal Volume overload Heart murmur

• 19
B is not correct. 5 0/o chose this.
• 20 Coarctation of the aorta is a congenital heart defect char.acterized by the narrowing of the aorta just distal to the subclavian
• 21 artery at the j unction of the ductus arteriosus and the aortic arch . This leads to decreased flow distal to the constriction and
• 22 increased flow proximal to it. Patients often present in childhood with asymptomatic hypertension due to decreased blood
flow to kidneys. On physica l exam, blood pressure in the upper extrem it ies is greater than in the lower extrem ities with weak
• 23
femoral and distal pu lses. Patients develop collateral vessels in the back to bypass the narrowing and a continuous murmu r
• 24 ... . .. .. . . . . ,, . . ..

Lock
s
Suspend
0
End Block
Item: 17 of 24 ~ 1 • Mark <:] (:>- Jill ~· ~J
QID: 21554 ..1 Previous Next Lab'V!I!ues Notes Calculator

1
B is not correct. 50/o chose this.
2
Coarctation of the aorta is a congenital heart defect characterized by the narrowing of the aorta just distal to the subclavian
3 artery at the junction of the ductus arteriosus and the aortic arch. This leads to decreased flow distal to the constriction and
4 increased flow proximal to it. Patients often present in childhood with asymptomatic hypertension due to decreased blood
5
flow to kidneys. On physical exam, blood pressure in the upper extremities is greater than in the lower extremities with weak
femora l and distal pulses. Patients develop collateral vessels in the back to bypass the narrowing and a continuous murmur
6
may be heard over these vessels . The first and second heart sounds are usually normal. There may be an ejection systolic
7 click and a systolic ejection murmur from coexisting cardiac defects such as bicuspid aortic valve or aortic stenosis.
8 Coarctation of the aorta Subclavian artery Congenital heart defect Bicuspid aortic valve Aortic arch Aorta Aortic valve Ductus arteriosus Hypertension Aortic stenosis

9 Congenital disorder Systole Heart sounds Blood pressure Stenosis Anatomical terms of location Mitral valve Kidney Heart murmur Physical examination Asymptomatic Femoral artery

10 C is not correct. 4% chose this.


11 Transposition of the great arteries (TGA) occurs when the aorta arises from the right ventricle and the pulmonary artery rises
12 from the left ventricle . Dextro-TGA (D-TGA) describes the form in which the aorta is anterior and to the right of the
13
pulmonary artery. It usua lly presents wit h cyanosis and tachypnea w ithin hours of birth because the systemic and pulmonary
circulations are parallel rather than serial. Oxygenated bllood circulates through the lungs repeatedly, while deoxygenated
14
blood is pumped to the body. D-TGA is incompatible w ith life unless there are other defects present, such as atrial or
15 ventricula r septal defects or a patent ductus arteriosus, that allow for mixing between the two parallel circulations and
16 delivery of oxygen to the body.
Cyanosis Patent ductus arteriosus Tachypnea Pulmonary artery Transposition of the great vessels Aorta Dextro-Transposition of the great arteries Ventricle {heart) Ductus arteriosus
17
Great arteries Artery Oxygen Anatomical terms of location Atrium (heart) Lung
• 18
• 19 D is not correct. 120/o chose this .
• 20 Tetralogy of Fallot (TOF) is the most common cyanotic congenital cardiac abnormality in children . It is composed of four
• 21 defects: right ventricula r outflow tract obstruction (pulmonary stenosis/atresia), ventricular septal defect (VSD), right
ventricula r hypertrophy, and an overriding aorta . TOF usually presents in infancy, with cyanotic spells, a normal Sl, single 52,
• 22
and a harsh systolic crescendo-decrescendo ejection murmur (due to right ventricu la r outflow tract obstruction) . This murmur
• 23 is heard best over the left upper sternal border and radiates to the back. It is also possible to have a holosystolic murmur at
• 24 th<> l<>ft lnw <>r <:t<>rn::.l hn rrl<>r fr,-,m th<> \/<:;n ::.nrl ::. r"nntin11noo<: m::.l"hin<>rv moor-moor if::. n::.t<>nt rlool"too<: ::.rt<>rin<:ot<: i<: nr<><:<>nt

Lock
s
Suspend
0
End Block
Item: 17 of 24 ~ 1 • Mark <:] (:>- Jill ~· ~J
QID: 21554 ..1 Previous Next Lab'V!I!ues Notes Calculator

1 D is not correct. 1 20/o chose this.


2 Tetralogy of Fallot (TOF) is the most common cyanotic congenita l cardiac abnormality in children . It is composed of four
3 defects: right ventricular outflow tract obstruction (pulmonary stenosis/atresia), ventricular septal defect (VSD), right
4
ventricular hypertrophy, and an overriding aorta . TOF usually presents in infancy, with cyanotic spells, a normal Sl, single 52,
and a harsh systolic crescendo-decrescendo ejection murmur (due to right ventricu lar outflow tract obstruction) . This murmur
5 is heard best over the left upper sternal border and radiates to the back. It is also possible to have a holosystolic murmur at
6 the left lower sternal border from the VSD and a continuous mach inery murmur if a patent ductus arteriosus is present.
Tetralogy of Fallot Ventricular septal defect Patent ductus arteriosus Overriding aorta Cyanotic heart defect Aorta Right ventricular hypertrophy Ductus arteriosus Cyanosis
7
Ventricular hypertrophy Systole Congenital disorder Hypertrophy Ventricular outflow tract Ventricular outflow tract obstruction Heart munnur Sternum
8
9 E is not correct. 15% chose this.
10 Ventricular septal defects (VSDs) are the most common congenital heart defect. Symptoms vary widely depending on the
11 degree of left-to-right shunt. Patients may be asymptomatic or present with congestive heart failure, delayed growth,
12
exercise and/or feeding intolerance, and frequent respiratory infections . On physical exam, there is a harsh, vibratory
holosystolic murmur at the lower left sternal border with a loud 52 . A diastolic rumble due to increased flow across the mitral
13 valve may also be heard .
14 Congenital heart defect Heart failure Congestive heart failure Mitral valve Congenital disorder Asymptomatic Diastole Ventricle (heart) Cardiac shunt Ventricular system

15 Physical examination Sternum Interventricular septum Heart munnur

16
17
Bottom line :
• 18
Atrial septal defects (ASDs) are often asymptomatic and cause acyanotic heart disease . Physical exam may be remarkable
• 19
for a wide, fixed, split 52 with a systolic ej ection murmur at the left upper sternal border.
• 20 Split 52 Systole Atrium (heart) Sternum Cardiovascular disease Asymptomatic Physical examination Heart murmur
• 21
• 22
• 23 References:
• 24
y FA Step 2 CK 9th ed p 373

Lock
s
Suspend
0
End Block
Item: 17 of 24 ~ 1 • Mark <:] (:>- Jill ~· ~J
QID: 21554 ..1 Previous Next Lab'V!I!ues Notes Calculator

1 Tetralogy of Fallot (TOF) is the most common cyanotic congenital cardiac abnormality in children . It is composed of fou r
2 defects: right ventricular outflow tract obstruction (pulmonary stenosis/atresia), ventricular septal defect (VSD), right
ventricular hypertrophy, and an overriding aorta . TOF usually presents in infancy, with cyanotic spells, a normal Sl, single 52,
3
and a harsh systolic crescendo-decrescendo ejection murmur (due to right ventricu lar outflow tract obstruction) . This murmur
4 is heard best over the left upper sternal border and radiates to the back. It is also possible to have a holosystolic murmur at
5 the left lower sternal border from the VSD and a continuous machinery murmur if a patent ductus arteriosus is present.
Tetralogy of Fallot Ventricular septal defect Patent ductus arteriosus Overriding aorta Cyanotic heart defect Aorta Right ventricular hypertrophy Ductus arteriosus Cyanosis
6
Ventricular hypertrophy Systole Congenital disorder Hypertrophy Ventricular outflow tract Ventricular outflow tract obstruction Heart murmur Sternum
7
8 E is not correct. 15% chose this.
9 Ventricular septal defects (VSDs) are the most common congenital heart defect. Symptoms vary widely depending on the
10 degree of left-to-right shunt. Patients may be asymptomatic or present with congestive heart failure, delayed growth,
exercise and/or feeding intolerance, and frequent respiratory infections . On physical exam, there is a harsh, vibratory
11
holosystolic murmur at the lower left sternal border with a loud 52 . A diastolic rumble due to increased flow across the mitral
12 valve may also be heard .
13 Congenital heart defect Heart failure Congestive heart failure Mitral valve Congenital disorder Asymptomatic Diastole Ventricle (heart) Cardiac shunt Ventricular system

14 Physical examination Sternum Interventricular septum Heart murmur

15
16
Bottom line:
17
Atrial septal defects (ASDs) are often asymptomatic and cause acyanotic heart disease . Physical exam may be remarkable
• 18
for a wide, fixed, split 52 with a systolic ejection murmur at the left upper sternal border.
• 19 Split 52 Systole Atrium (heart) Sternum Cardiovascular disease Asymptomatic Physical examination Heart murmur

• 20
• 21
• 22 References:
FA Step 2 CK 9th ed p 373
• 23
FA Step 2 CK 8th ed p 357
• 24

Lock
s
Suspend
0
End Block
Item: 18 of 24 ~ 1 • Mark <:] (:>- Jill ~· ~J
QID: 233 69 ..1 Previous Next Lab'V!I!ues Notes Calculator

1 •
A girl is delivered at 37 weeks' gestation v ia Pitocin-induced vaginal delivery for intrauterine growth restriction . The
2 pregnancy was uncomplicated, but prenatal ult rasounds throughout t he second and third trimesters measured long bone
3 length in the third to fifth percentile and weight in the fiftlh percentile for gestational age. Amniocentesis was performed
4 and the child was found to have the genotype 4S,X. At delivery, the child has a heart rate of 140/min and respiratory rate of
40/min . Physical exam ination reveals clear lungs bilaterally, an 53 heart sound, delayed femoral pulses, and lower acrocyanosis.
5
The child is transferred t o the neonatal intensive care unit for further management.
6
7
Which of the following is the most appropriat e next step in management?
8
9
A. Balloon angioplasty
10
11
B. Echocardiogram
12 C. Emergency surgical repair
13
D. Emergent intubation
14
15
E. Prostag landin E1 intravenous infusion
16
17
• 18
• 19
• 20
• 21
• 22
• 23
• 24

Lock
s
Suspend
0
End Block
Item: 18 of 24 ~ 1 • Mark <:] (:>- Jill ~· ~J
QID: 233 69 ..1 Previous Next Lab'V!I!ues Notes Calculator

2 The correct answer is E. 43% chose this.


3 The child described has Turner syndrome ( 4S,X) with associated coa rctation of the aorta . An important step in management
after birth is to determine the extent of coarctation, which is done by echocardiogram . However, until an echocardiogram can
4
be arran ged, the ductus arteriosus must be kept patent to facilitate blood flow past the coa rctation . If the cond ition is severe,
5 blood flow to the abdominal organs may be insufficient without a patent ductus arteriosus (PDA) . A PGE 1 analog w ill maintain
6 patency of the ductus arteriosus as a temporizing measure. Once the severity is determined, the decision to proceed with
7 either surgical or medical management may be made . About 30% of children with Turner synd rome have aortic va lve disease,
while up to 30% have a coa rctation of aorta . Children with Turner syndrome are often small for gestational age and have
8
slightly dysmorph ic featu re s including cubitus valgus, webbed neck, and w ide-set nipples .
9 Coarctation of the aorta Patent ductus arteriosus Turner syndrome Cubitus valgus Echocardiography Aorta Aortic valve Webbed neck Ductus arteriosus Gestational age

10 Valgus deformity Dysmorphic feature Stenosis


11
A is not correct. 110/o chose this.
12
Balloon angioplasty is an older method of coarctation repair to widen the narrowing in the aorta . It is rarely used now due to
13 the common complication of aortic aneurysm foll owing angioplasty. Balloon angioplasty with stent graft may still be used in
14 emergent situations and critically ill infants.
Stent Stent graft Angioplasty Aorta Aortic aneurysm Aneurysm Coarctation of the aorta
15
16 B is not correct. 250/o chose this.
17 An echocardiogram is required for this patient; however, as the patient is symptomatic she requires immediate treatment.
18 Once the patient is stabilized, imaging w ill be important to determine the extent of the defect.
Echocardiography
• 19
• 20 C is not correct. 140/o chose this .
• 21 Surgical repair of aortic coarctation is stil l considered gold standard t reatment but it is rarely done on an emergent basis, as
medical treatment is effective in ensuring lower extremity perfusion while preparing for surgery. I mpro ved surgical techniques
• 22
in coa rctation repa ir and reduced risk of aneurysm ( as compared to the risk of aneurysm associated with balloon ang ioplasty)
• 23 have mostly replaced balloon angioplasty repair.
• 24

Lock
s
Suspend
0
End Block
Item: 18 of 24 ~ 1 • Mark <:] (:>- Jill ~· ~J
QID: 23369 ..1 Previous Next Lab'V!I!ues Notes Calculator
• I I • • I I ..
1
An echocardiogram is required for th is patient; however, as the patient is symptomatic she requires immediate t reatment.
2
Once the pati ent is stabilized, imaging will be important to determine the extent of the defect.
3 Echocardiography
4
C is not correct. 140/o chose this.
5
Surgical repair of aortic coarctation is stil l considered gold standard treatment but it is rarely done on an emergent basis, as
6 medical t reatment is effective in ensuring lower extremity perfusion while preparing for surgery. Improved surgical techniques
7 in coarctation repa ir and reduced risk of aneurysm (as compared to the risk of aneurysm associated with balloon angioplasty)
8 have mostly replaced balloon angioplasty repair.
Coarctation of the aorta Angioplasty Gold standard (test) Perfusion Aneurysm Stenosis
9
10 D is not correct. 7 % chose this.
11 Emergent intubation is not necessary at this time, as the child has a stable airway and is in no respiratory distress.
Transferring the patient to neonatal intensive care unit makes this option available, in case patient was to develop respiratory
12
distress.
13 Neonatal intensive care unit Intensive care unit Intubation Dyspnea Respiratory distress Intensive care medicine Infant Respiratory tract
14

15
Bottom Line :
16
17
Forty-five percent of children w ith Turner syndrome have cardiac defects, and of these, coarctation of the aorta is the most
common . Children with Turner syndrome are often small for gestational age with slightly dysmorphic features including
18 cubitus valgus, webbed neck, and w ide-set nipples .
• 19 Cubitus valgus Turner syndrome Coarctation of the aorta Webbed neck Aorta Small for gestational age Valgus deformity Gestational age Valgus Oysmorphic feature

• 20
• 21
• 22 References:
FA Step 2 CK 9th ed pp 334; 339; 375
• 23
FA Step 2 CK 8th ed p 363
• 24

Lock
s
Suspend
0
End Block
Item: 19 of 24 ~ 1 • Mark <:] (:>- Jill ~· ~J
QID: 23098 ..1 Previous Next Lab'V!I!ues Notes Calculator

1 •
A 1-week-old child who has developed r espiratory distress has higher blood pressure in his upper extremities than in his ~~AI
2 lower extremities. On radiolog ic examination there is constriction of his aorta j ust proximal to where the ductus arteriosus
3 is in communication with the aorta .
4
5 Which of the following is the appropriate init ial management?
6
7 A. Antibiotics
8
B. Coil embolization of the patent ductus arteriosus
9
10
C. Indomethacin administration

11 D. Open sur gical closu re of the patent ductus arteriosus


12
E. Prostaglandin E1 administration
13
14
15
16
17
18
• 19
• 20
• 21
• 22
• 23
• 24

Lock
s
Suspend
0
End Block
Item: 19 of 24 ~ 1 • Mark <:] (:>- Jill ~· ~J
QID: 23098 ..1 Previous Next Lab'V!I!ues Notes Calculator

2 The correct answer i s E. 700/o chose this.


3 This ch ild has a preductal aortic coarctation . The obstruction of the aorta has increased t he afterload on the left ventricle and
resulted in congestive heart fa ilure, manifested as respiratory distress . Because the obstruction is proximal to the entry of t he
4
ductus arteriosus, blood flow to the abdomen and lower extremities has been supplied primarily by the ductus arteriosus and
5 collateral flow th rough the intercosta l arteries . As the ductus arteriosus begins to close, however, a sign ificant fraction of the
6 blood supply to the lower body is impaired . Although definitive surgical repair of the coarctation is necessary for long-term
7
survival, administration of prostag landin E1 will help maintain t he patency of the ductus arteriosus, a temporizing measure
until defin itive repair.
8 Coarctation of the aorta Afterload Prostaglandin Heart failure Aorta Congestive heart failure Ventricle (heart) Ductus arteriosus Artery Anatomical terms of location Intercostal arteries
9 Abdomen Dyspnea Stenosis
10
A i s not correct. 2 % chose this.
11
Endocardit is, or more appropriately, endarteritis, can occu r at the site of coa rctation . Therefore, endocarditis prophylaxis with
12
antibiotics is appropriate in these pati ents. Because the ch ild is in r espiratory distress and at imminent risk for hypoperfusion
13 of his abdomen and lower extremities, maintain ing patency of the ductus arteriosus is of primary importance over reducing
14 the risk of endocarditis/endoarteritis.
Endocarditis Ductus arteriosus Dyspnea Shock (circulatory) Antibiotics Abdomen Preventive healthcare Coarctation of the aorta Respiratory distress Stenosis
15
16 B is not correct. 3 0/o chose this.
17 Along with surgical closure and indomethacin administration, coil embolization is a third, m inimally invasive way to close the
18
ductus arteriosus . However, continued patency, not closure of the ductus arteriosus, is the desired effect.
lndometacin Ductus arteriosus Embolization
19
• 20 Cis not correct. 210/o chose this .
• 21 Patency of the ductus arteriosus is important for survival in this child . Although prostaglandins help maintain patency,
indomethacin (a prostagland in inhibitor) promotes the closu re of the ductus arteriosus, the opposite of what is needed .
• 22
lndometacin Ductus arteriosus Prostaglandin
• 23
• 24
D is not correct. 4 % chose this .

Lock
s
Suspend
0
End Block
Item: 19 of 24 ~ 1 • Mark <:] (:>- Jill ~· ~J
QID: 23098 ..1 Previous Next Lab'V!I!ues Notes Calculator

1 • t he risk of endocardit is/endoarteritis.


Endocarditis Ductus arteriosus Dyspnea Shock (circulatory) Antibiotics Abdomen Preventive healthcare Coarctation of the aorta Respiratory distress Stenosis
2
3 B is not correct. 30/o chose this .
4 Along w ith surgical closure and indomethacin administration, coil embolization is a third, m inimally invasive way to close the
5 ductus arteriosus . However, continued patency, not closure of the ductus arteriosus, is the desired effect.
Indometadn Ductus arteriosus Embolization
6
7 Cis not correct. 2 10/o chos e this .
8 Patency of the ductus arteriosus is important for survival in this child . Although prostagland ins help maintain patency,
indomethacin (a prostaglandin inhibitor) promotes the closu re of the ductus arteriosus, the opposite of what is needed .
9 Indometadn Ductus arteriosus Prostaglandin
10
D is not correct. 4 % chos e this .
11
Surgical closu re of the patent ductus arteriosus is analogous to indomethacin admin istration; both will lead to rapid
12
decompensation in this patient because perfusion to the lower half of the body is reliant in part on the patency of the ductus
13 arteriosus .
14 Patent ductus arteriosus Indometacin Ductus arteriosus Patent Perfusion

15
16
Bottom line :
17
Administration of prostaglandin E1 will help maintain the patency of the ductus arteriosus, a temporizing measure until
18 definitive repair of preductal aortic coarctation .
19 Coarctation of the aorta Prostaglandin Ductus arteriosus

• 20
• 21
• 22 References:
FA Step 2 CK 9th ed pp 22-24
• 23
FA Step 2 CK 8th ed pp 24; 25; 28
• 24

Lock
s
Suspend
0
End Block
Item: 20 of 24 ~ 1 • Mark <:] (:>- Jill ~· ~J
QID: 23648 ..1 Previous Next Lab'V!I!ues Notes Calculator

1 •
A 17-year-old boy presents to his pediatrician complaining of excess fatigue when playing basketball that has been
2 worsen ing over the past several months. He has also had multiple "colds" over the same t ime frame, but has been
3 otherwise hea lthy. He is afebri le with a heart rate of 75/min and blood pressure of 112/72 mm Hg . His lungs are clear to
4 auscultation, but on cardiac examination he has a wide, fi xed split 5 2 and a systolic ejection murmur at the left upper sternal
border. X-ray of the chest shows his heart is slightly en larged, and there are significant pulmonary vascu lar markings .
5
6
For wh ich of the following is the patient at greatest risk later in life?
7
8
A. Arrhythmias
9
10 B. Bacteria l endocarditis
11 C. Left-sided heart failure
12
D. Myocard ial infarction
13
14 E. Systemic hypertension
15
16
17
18
19
• 20
• 21
• 22
• 23
• 24

Lock
s
Suspend
0
End Block
Item: 20 of 24 ~ 1 • Mark <:] (:>- Jill ~· ~J
QID: 23648 ..1 Previous Next Lab'V!I!ues Notes Calculator

1 •
2 The correct answer is A. 41% chose this.
3 The wide, fixed split 5 2 and the systolic ejection mu rmur are due to increased blood flow across the pulmonic valve from a
4 left-to-right shunt secondary to an atrial septa l defect (ASD) . A symptomatic, uncorrected ASD makes him prone to
5
tachyarrhythmias later in life, as well as Eisenmenger's syndrome (pulmonary hypertension causing the shunt to become
right to left) . He is also more likely to have respiratory infections .
6 Atrial septal defect Pulmonary valve Pulmonary hypertension Eisenmenger's syndrome Systole Hypertension Blood flow Cardiac shunt Shunt (medical) Heart murmur Atrium (heart)
7
B is not correct. 150/o chose th is.
8
Patients with ASD are not at significantly increased risk of bacterial endocardit is.
9 Endocarditis Infective endocarditis
10
C is not correct. 280/o chose this.
11
This patient has an AS D and a left-to-right shunt . Eventually, this will cause pulmonary hypertension and right ventricular
12
hypertrophy and potentially failure, as opposed to left sided .
13 Pulmonary hypertension Ventricular hypertrophy Hypertension Right ventricular hypertrophy Hypertrophy Cardiac shunt Shunt (medical)
14
D i s not correct. 80/o chose this.
15
This patient has an ASD and a left-to-right shunt. This does not affect the development of a myoca rdial infarction .
16 Myocardial infarction Shunt (medical)
17
E i s not correct. 8 0/o chose th is.
18
This patient has an AS D and a left-to-right shunt . This does not affect the development of systemic hypertension .
19 Hypertension Cardiac shunt Shunt (medical)
20
• 21
• 22
Bottom line :
• 23 A symptomatic, uncorrected atrial septal defect makes patients prone to tachyarrhythmias later in life, as well as
Eisenmenger's syndrome (pulmonary hypertension causing the shunt to become right to left) .
• 24

Lock
s
Suspend
0
End Block
Item: 20 of 24 ~ 1 • Mark <:] (:>- Jill ~· ~J
QID: 23648 ..1 Previous Next Lab'V!I!ues Notes Calculator

1 Atrial septal defect Pulmonary valve Pulmonary hypertension Eisenmenger's syndrome Systole Hypertension Blood flow Cardiac shunt Shunt (medical) Heart murmur Atrium (heart)

2
B is not correct. 150/o chose this.
3 Patients with ASD are not at significantly increased risk of bacterial endocarditis.
4 Endocarditis Infective endocarditis

5
C is not correct. 280/o chose this.
6
This patient has an ASD and a left-to-right shunt. Eventually, this will cause pu lmonary hypertension and right ventricular
7 hypertr ophy and potentially failure, as opposed to left sided .
8 Pulmonary hypertension Ventricular hypertrophy Hypertension Right ventricular hypertrophy Hypertrophy Cardiac shunt Shunt (medical)

9 D is not correct. 8% chose this.


10 This patient has an ASD and a left-to-right shunt. This does not affect the development of a myocardial infarction .
11 Myocardial infarction Shunt (medical)

12 E is not correct. 80/o chose this.


13 This patient has an ASD and a left-to-right shunt. This does not affect the development of systemic hypertension .
14 Hypertension Cardiac shunt Shunt (medical)

15
16
Bottom line:
17
A symptomatic, uncorrected atrial septa l defect makes patients prone to tachyarrhythmias later in life, as wel l as
18 Eisenmenger's syndrome ( pulmonary hypertension causing the shunt to become right to left).
19 Atrial septal defect Pulmonary hypertension Eisenmenger's syndrome Hypertension Shunt (medical) Atrium (heart) Cardiac arrhythmia

20
• 21
• 22 References:
FA Step 2 CK 9th ed pp 372-373; 376
• 23
FA Step 2 CK 8th ed p 357
• 24

Lock
s
Suspend
0
End Block
Item: 21 of 24 ~ 1 • Mark <:] (:>- Jill ~· ~J
QID: 24044 ..1 Previous Next Lab'V!I!ues Notes Calculator

1 •
A child is born to a healthy mother at 38 weeks' gestatiorn . The mother received no prenatal ca re but cla ims to have taken ~~AI
2 v itamins throughout her pregnancy. The child is noted to have holoprosencepha ly, microphtha lmia, polydactyly,
3 ompha locele, and mid li ne cleft li p.
4
5 Which of the following is the most likely diagnosis?
6
7 A. Neura l tube defect
8
B. Rubella infection
9
10
C. Trisomy 13

11 D. Trisomy 18
12
E. Trisomy 21
13
14
15
16
17
18
19
20
• 21
• 22
• 23
• 24

Lock
s
Suspend
0
End Block
Item: 21 of 24 ~ 1 • Mark <:] (:>- Jill ~· ~J
QID: 24044 ..1 Previous Next Lab'V!I!ues Notes Calculator

2 The correct answer is C. 690/o chose this .


3 This ch ild has trisomy 13, or Patau's syndrome. The major midline dysmorphic features of trisomy 13 are due to a defect in
4
the fusion of the midli ne precordia l mesoderm in the first 3 weeks of gestation, and may include mental and motor
retardation, polydactyly, microcephaly, low-set ears, holoprosencephaly, heart defects, m icrophthalmia, midline cleft lip
5 and/or palate, and omphalocele . Most children affected with trisomy 13 die w ithin the first 6 months.
6 Holoprosencephaly Omphalocele Patau syndrome Microcephaly Polydactyly Mesoderm Microphthalmia Cleft lip and palate Trisomy Lip Low-set ears Palate Gestation

7 Dysmorphic feature Congenital heart defect

8
A i s not correct. 9 % chose this.
9
Neural tube defects are abnormalities of the brain and spinal cord . They are associated with folic acid deficiency. Several
10 organ systems are affected in this child, wh ich should raise suspicion for an underlying syndrome.
11 Folic acid Neural tube Neural tube defect Spinal cord Brain Folate deficiency

12 B is not correct. 5 0/o chose this.


13 Congenital rubella infection can have devastating effects on the newborn. The most common cl inical findings are
14 sensorineural deafness, ophthalmologic abnormalities (especially cataracts), cardiac manifestations including patent ductus,
15
and meningoencephalitis.
Congenital disorder Sensorineural hearing loss Rubella cataract Hearing loss Ophthalmology Congenital rubella syndrome Meningoencephalitis Infection
16
17 D i s not correct. 150/o chose this .
18
Trisomy 18, or Edwards' syndrome, is the second most common trisomy observed in live births. Cli nical manifestations
include feeding difficulties, hypotonia eventually leading to hypertonia, prominent occiput, m icrognathia, horseshoe kidneys,
19
flexed fingers with overlapping fingers, congenital heart defects, and malrotation of the gastrointestinal system .
20 Edwards syndrome Hypertonia Micrognathism Hypotonia Occiput Horseshoe kidney Gastrointestinal tract Trisomy Congenital heart defect Intestinal malrotation Congenital disorder

21 Kidney Dysphagia
• 22
E i s not correct. 2 0/o c hose th is .
• 23
Trisomy 21, or Down's syndrome, is the most common trisomy found in live births . Clinical man ifestations include upslanting
• 24 ... • • • • • • ... ' • " • 0 • .. • ,, .. •

Lock
s
Suspend
0
End Block
Item: 21 of 24 ~ 1 • Mark <:] (:>- Jill ~· ~J
QID: 24044 ..1 Previous Next Lab'V!I!ues Notes Calculator

1
Congenital rubella infection can have devastating effects on the newborn . The most common clinical findings are
2 sensorineural deafness, ophthalmologic abnormalities (especially cataracts), cardiac manifestations including patent ductus,
3 and meningoencephalitis.
Congenital disorder Sensorineural hearing loss Rubella Cataract Hearing loss Ophthalmology Congenital rubella syndrome Meningoencephalitis Infection
4

5 D is not correct. 150/o chose this.


6 Trisomy 18, or Edwards' syndrome, is the second most common trisomy observed in live births. Clinical manifestations
7
include feeding difficu lt ies, hypotonia eventua lly leading to hypertonia, prominent occiput, micrognath ia, horseshoe kidneys,
flexed fingers with overlapping fingers, congenital heart defects, and malrotation of the gastrointestinal system.
8 Edwards syndrome Hypertonia Micrognathism Hypotonia Occiput Horseshoe kidney Gastrointestinal tract Trisomy Congenital heart defect Intestinal malrotation Congenital disorder
9 Kidney Dysphagia
10
E is not correct. 20/o chose this.
11
Trisomy 21, or Down's syndrome, is the most common trisomy found in live births . Clinical manifestations include upslanting
12
palpebral fissures, epicanthal folds, small mouth with a protrud ing tongue, simian (transverse) palmar crease, mental
13 retardation, flat nasal bridge, and excessive skin at the nape of the neck.
14 Down syndrome Single transverse palmar crease Palpebral fissure Epicanthic fold Trisomy Nasal bridge Intellectual disability Eyelid

15
16
Bottom line:
17
Trisomy 13 presents with a number of m idli ne defects, wh ich distinguish it from trisomy 18. In general, defects in babies
18 w ith trisomy 13 are more severe.
19 Patau syndrome Edwards syndrome Trisomy

20
21
• 22 References:
FA Step 2 CK 9th ed p 381
• 23
FA Step 2 CK 8th ed p 363
• 24

Lock
s
Suspend
0
End Block
Item: 21 of 24 ~ 1 • Mark <:] (:>- Jill ~· ~J
QID: 24044 ..1 Previous Next Lab'V!I!ues Notes Calculator

1
Congenital rubella infection can have devastating effects on the newborn . The most common clinical findings are
2 sensorineural deafness, ophthalmologic abnormalities (especially cataracts), cardiac manifestations including patent ductus,
3 and meningoencephalitis.
Congenital disorder Sensorineural hearing loss Rubella Cataract Hearing loss Ophthalmology Congenital rubella syndrome Meningoencephalitis Infection
4

5 D is not correct. 150/o chose this.


6 Trisomy 18, or Edwards' syndrome, is the second most common trisomy observed in live births. Clinical manifestations
7
include feeding difficu lt ies, hypotonia eventua lly leading to hypertonia, prominent occiput, micrognath ia, horseshoe kidneys,
flexed fingers with overlapping fingers, congenital heart defects, and malrotation of the gastrointestinal system.
8 Edwards syndrome Hypertonia Micrognathism Hypotonia Occiput Horseshoe kidney Gastrointestinal tract Trisomy Congenital heart defect Intestinal malrotation Congenital disorder
9 Kidney Dysphagia
10
E is not correct. 20/o chose this.
11
Trisomy 21, or Down's syndrome, is the most common trisomy found in live births . Clinical manifestations include upslanting
12
palpebral fissures, epicanthal folds, small mouth with a protrud ing tongue, simian (transverse) palmar crease, mental
13 retardation, flat nasal bridge, and excessive skin at the nape of the neck.
14 Down syndrome Single transverse palmar crease Palpebral fissure Epicanthic fold Trisomy Nasal bridge Intellectual disability Eyelid

15
16
Bottom line:
17
Trisomy 13 presents with a number of m idli ne defects, wh ich distinguish it from trisomy 18. In general, defects in babies
18 w ith trisomy 13 are more severe.
19 Patau syndrome Edwards syndrome Trisomy

20
21
• 22 References:
FA Step 2 CK 9th ed p 381
• 23
FA Step 2 CK 8th ed p 363
• 24

Lock
s
Suspend
0
End Block
Item: 22 of 24 ~ 1 • Mark <:] (:>- Jill ~· ~J
QID: 22443 ..1 Previous Next Lab'V!I!ues Notes Calculator

1 •
A 15-day-old full-term boy is brought to the clin ic by his parents because of shortness of breath, fail ure to tolerate ~~AI
2 feeding, and inadequate weight gain . On physica l exam ination there is no cyanosis evident on mucosal membranes.
3 Pulmonary ra les are heard bilaterally with an active preco,rdium and a soft 2/6 holosystolic murmur most prominent at the
4 left lower sternal border. ECG is significant for biventricular hypertrophy. X-ray of the chest is shown in the image .
5
6
7
8
9
10
11
12
13
14
15
16
17
18
19
20
Which of the following is the most likely etiology?
21
• 22 A. Aortic stenosis
• 23 B. Patent ductus arteriosus
• 24

Lock
s
Suspend
0
End Block
14
Which of the following is the most likely etiology?
15
16
A. Aortic stenosis
17
18 B . Patent ductus arteriosus
19 C. Tetralogy of Fallot
20
D . Transposition of the great arteries
21
• 22 E. Ventricular septal defect
• 23

• 24

a
Lock
s
Suspend
8
End Block
Item: 22 of 24 ~ 1 • Mark <:] (:>- Jill ~· ~J
QID: 22443 ..1 Previous Next Lab'V!I!ues Notes Calculator

2
The correct answer is E. 520/o chose this.
When evaluating heart defects, it is important to consider the presence or absence of cyanosis, nature of any murmur, and
3
age at presentation . Ventricular septal defects (VSDs) are the most common congenital heart defects and account for most
4 hospital admissions for heart defects in infants 14-28 days old. The nature of symptoms is highly dependent on the size of
5 the defect. The ECG findings and the quiet murmur in thi s child are consistent with a large VSD. Radiographic findings
6 commonly include increased pulmonary vascular markings as seen in this patient. In more severe VSDs, enlargement of the
left atrium and left ventricle as well as the pulmona ry artery can also be seen on x-ray of the chest.
7
Cyanosis Pulmonary artery Ventricle {heart) Atrium (heart) Electrocardiography Congenital heart defect Congenital disorder Ventricular system X-ray Radiography
8
Interventricular septum Heart murmur Blood vessel
9
10
A is not correct. 6% chose this.
Aortic stenosis is usually the result of a bicuspid or single commissure valve and symptoms ar e largely dependent on the
11
degree of stenosis. ECG would likely only show left ventricular hypertrophy. Most importantly, ventricular septal defects would
12 be more common at this age.
13 Aortic stenosis left ventricular hypertrophy Ventricular hypertrophy Stenosis Electrocardiography Hypertrophy Mitral valve Heart

14
B is not correct. 60/o chose this.
15 Patent ductus arteriosus is often seen in prematur e infants and carries a female predominance. The characteristic murmur is
16 often described as machine-like and involves both systole and diastole.
Patent ductus arteriosus Systole Diastole Ductus arteriosus Preterm birth
17
18 C is not correct. 250/o chose this.
19 Tetralogy of Fallot is the second most common heart defect in infants 14-28 days old, but would likely present with cyanosis.
20 Additionally, pulmonary blood flow would be decreased in this condition . Remember, the tetralogy includes right ventricular
outflow obstruction, right ventricular hypertrophy, overri ding aorta , and subaortic ventricular septal defect. Radiographic
21
findings typically include a boot-shaped heart due to hypertrophy of the right ventricle .
22 Tetralogy of Fallot Ventricular septal defect Cyanosis Overriding aorta Ventricular hypertrophy Aorta Hypertrophy Right ventricular hypertrophy Ventride (heart) Congenital heart defect
• 23 Blood flow
• 24

Lock
s
Suspend
0
End Block
Item: 22 of 24 ~ 1 • Mark <:] (:>- Jill ~· ~J
QID: 22443 ..1 Previous Next Lab'V!I!ues Notes Calculator

1 B is not correct. 6 0/o chose this .


2 Patent duct us arteriosus is often seen in prematur e infants and carries a female predominance. The characteristic murmur is
3 often described as machine-like and involves both systole and diastole.
Patent ductus arteriosus Systole Diastole Ductus arteriosus Preterm birth
4

5 C is not correct. 250/o chose this.


6 Tetralogy of Fallot is the second most common heart defect in infants 14-28 days old, but would likely present with cyanosis.
7
Additionally, pulmonary blood flow would be decreased in this condition . Remember, the tetralogy includes right ventricular
outflow obstruction, right ventricular hypertrophy, overri ding aorta , and subaortic ventricular septal defect. Radiographic
8
findings typically include a boot-shaped heart due to hypertrophy of the right ventricle .
9 Tetralogy of Fallot Ventricular septal defect Cyanosis Overriding aorta Ventricular hypertrophy Aorta Hypertrophy Right ventricular hypertrophy Ventricle !heart) Congenital heart defect

10 Blood flow

11
Dis not correct. 110/o chose this .
12
Transposition of the great arteries is also a common heart defect, but cyanosis would likely be present from birth . Other
13 features are variable but incr eased pu lmonary vascular fi nd ings are typical.
14 Cyanosis Transposition of the great vessels Great arteries Artery Congenital heart defect Blood vessel

15
16
Bottom line :
17
VSDs are the most common noncyanotic congenital heart defects. A crit ical VSD may present with failure to thrive and
18 respiratory distress.
19 Failure to thrive Congenital disorder Dyspnea Congenital heart defect Respiratory distress

20
21
22
References:
FA Step 2 CK 9th ed pp 22-28
• 23
FA Step 2 CK 8th ed pp 21-28
• 24

Lock
s
Suspend
0
End Block
Item: 23 of 24 ~ 1 • Mark <:] (:>- Jill ~· ~J
QID: 21545 ..1 Previous Next Lab'V!I!ues Notes Calculator

1 •
A cardiologist is called to consult on the ca re of a 2-day-old girl delivered at 33 weeks' gestation . The infant is lying supine ~~AI
2
in her isolette. She is acyanotic, but has a heart rate of 192/min and a respiratory rate of 60/min . She has a nonradiating
3 continuous machinery murmur at the left upper sternal border that remains the same with compression of the ipsilateral,
4 then contra lateral jugular veins. 5 1 and 52 are normal. Her periphera l pulses are bound ing .
5
6 Which of the following is the most likely diagnosis?
7
8 A. Aortic stenosis with aortic regurgitation
9 B. Patent ductus arteriosus
10
C. Systemic arteriovenous fistula
11
12 D. Venous hum
13 E. Ventricular septal defect
14
15
16
17
18
19
20
21
22
• 23
• 24

Lock
s
Suspend
0
End Block
Item: 23 of 24 ~ 1 • Mark <:] (:>- Jill ~· ~J
QID: 21545 ..1 Previous Next Lab'V!I!ues Notes Calculator

1
The correct answer is B. 900/o chose this.
2
Patent ductus arteriosus (PDA) is a vascular connection that exists between the aorta and main pulmonary artery. It causes
3
acyanotic congenital heart disease. PDAs account for 10% of congenital heart disease, occurring w ith a high incidence in
4 preterm infants and a 2 : 1 female : male predom ina nee . Sm a II PDAs are asymptomatic, while large ones may cause congestive
5 heart failure, failure to thrive, and recurrent lower respiratory tract infections . On physical examination, a continuous
6
machinery murmur heard best at the left upper sternal border, and bounding peripheral pulses may be present. There may
also be a prominent apical impulse and a thrill. X-ray of the chest may show cardiomegaly and increased pulmonary vascular
7 markings. PDAs usually close with in the first month of life . I n preterm infants, indomethacin may be administered to close the
8 PDA. PDAs also may be surgically ligated or coil embolized if necessary.
Patent ductus arteriosus Indometacin Cardiomegaly Heart failure Pulmonary artery Congenital heart defect Congestive heart failure Chest radiograph Aorta Embolism Failure to thrive
9
10 Ductus arteriosus Asymptomatic Blood vessel Congenital disorder Cardiovascular di sease X-ray Embolization Acyanotic heart defect Heart murmur Physical examination

11 A is not correct. 4% chose this.


12 The murmur of aortic stenosis with aortic regurg itation is loudest over the right sternal border and radiates to the neck and
13 apex. The murmur is usually preceded by an ejection click heard best at the left lower sternal border.
Aortic insufficiency Aortic stenosis Stenosis Regurgitation (circulation) Sternum Heart murmur Regurgitation (digestion)
14

15 C is not correct. 2% chose this.


16 Systemic arteriovenous fistulas are usually extracardiac in location and can produce continuous murmurs. However, the
17
murmur would not be expected to occur in the location described . The rest of the physical examination findings are also most
consistent w ith a PDA.
18 Continuous murmurs Heart murmur
19
D is not correct. 1 Ofo chose this.
20
The continuous murmur of venous hum is usually loudest on the right (but can also be present on the left) supra- and
21
infraclavicula r areas. It usually is inaudible with the patient in the supine position, but intensifies when the patient is sitting .
22 It also disappears w ith compression of the ipsilatera l jugular vein . It is an innocent murmur that is produced by alteration in
23 blood flow through veins .
Supine position Jugular vein Heart murmur Venous hum Anatomical terms of location
• 24

Lock
s
Suspend
0
End Block
Item: 23 of 24 ~ 1 • Mark <:] (:>- Jill ~· ~J
QID: 21545 ..1 Previous Next Lab'V!I!ues Notes Calculator
I I - • f I •
1
Systemic arteriovenous fistulas are usually extracard iac in location and can produce continuous murmurs. However, the
2
murmur would not be expected to occur in the location described . The rest of the physical examination findings are also most
3 consistent w ith a PDA.
4 Continuous murmurs Heart murmur

5 Dis not correct. 1% chose this.


6 The continuous murmu r of venous hum is usually loudest on the right (but can also be present on the left) supra- and
7 infraclavicular areas. It usually is inaudible with the patient in the supine position, but intensifies when the patient is sitting .
8 It also disappears with compr ession of the ipsilateral jugular vein. It is an innocent murmur that is produced by alteration in
blood flow through veins .
9 Supine position Jugular vein Heart murmur Venous hum Anatomical terms of location
10
E i s n ot correct. 30/o chose t his.
11
Ventricular septal defects are the most common congenital heart defects. They usually ar e associated w ith a holosystolic
12
murmur that is loudest at the left lower sternal border.
13 Ventricular system Congenital heart defect Sternum Congenital disorder Ventricle (heart) Interventricular septum Heart murmur Holosystolic
14

15
Bottom Li ne:
16
17
Patent ductus arteriosus, w hich causes acyanotic congenital heart disease, is a vascular connection between the aorta and
main pulmonary artery. On physical exam a continuous machinery murmur is heard , bounding peripheral pulses are present,
18 and there may also be a prom inent apical impulse and a thrill.
19 Patent ductus arteriosus Pulmonary artery Aorta Congenital heart defect Ductus arteriosus Congenital disorder Acyanotic heart defect Cardiovascular disease Heart murmur Blood vessel

20
21
22
References:
FA Step 2 CK 9th ed pp 371-378
23
FA Step 2 CK 8th ed pp 355-360
• 24

Lock
s
Suspend
0
End Block
Item: 24 of 24 ~ 1 • Mark <:] (:>- Jill ~· ~J
QID: 21551 ..1 Previous Next Lab'V!I!ues Notes Calculator

2 A 1-year- old boy presents to a pediatric ca rdiologist for a follow- up visit. He is the product of a normal pregnancy and was
delivered at 37 weeks' gestation . He was referred to the pediatric cardiologist by his pediatrician 2 months ago when she
3
found a wide fixed 5 2 and 2/6 systolic ej ection murmur at the left upper sternal border. At that time he was acyanotic and
4 had no associated symptoms . An echocardiogram revealed mild cardiomegaly and a small atrial septal defect. Today he appears
5 well-nourished, acyanotic, and in no acute distress. His temperature is 37°C (98.6°F), pulse is 135/min, blood pressure is 90/56
6
mm Hg, and respiratory rate is 24/m in. The murmur is as described . His peripheral pulses are normal, and no clubbing of his
digits is noted . His lungs are clear to auscu ltation bilaterally. An echocardiogram is repeated and the findings are unchanged .
7
8
Which of the following is the best next step in management?
9
10
A. Antibiotic prophylaxis for bacterial endocardit is
11

12 B. Evaluation with a Holter mon itor


13 C. Monitor the patient for appropriate growth and signs of increased shunting
14
D. Percutaneous transcatheter closu re of the defect
15
16 E. Surgical closu re of the defect
17
18
19
20
21
22
23
• 24

Lock
s
Suspend
0
End Block
Item: 24 of 24 ~ 1 • Mark <:] (:>- Jill ~· ~J
QID: 21551 ..1 Previous Next Lab'V!I!ues Notes Calculator

2
The correct answer is C. 690/o chose this .
Most small atrial septal defects (ASDs) <8 mm in diameter close spontaneously within the first 2 years of life . Therefore,
3
surgery or t ranscatheter closure of an ASD is not recommended unless the patient is symptomatic or has a high rate of shunt
4 flow.
5 Shunt (electrical) Shunting (rail) Shunt (medical)

6
A i s not correct. 3% chose this .
7 Antibiotic prophylaxis for bacterial endocardit is is no longer recommended, except when the ASD is associated with mitral
8 regu rgitation .
Antibiotics Endocarditis Mitral insufficiency Infective endocarditis Preventive healthcare Antibiotic prophylaxis Regurgitation (circulation)
9
10 B is not correct. 2 0/o chose this .
11 The child does not have signs or symptoms of an arrhythmia such as pa lpitations, irregular heart rate, or syncope, and does
12 not need Holter monitoring .
Syncope (medicine) Cardiac arrhythmia Palpitations Heart rate
13
14 D is not correct. 150/o chose this .
15 Percutaneous closure of the defect is only used in symptomatic patients with an ostium secundum ASD. It is not an
16
appropriate treatment for ostium primum or sinus venosus ASDs.
Primary interatrial foramen Sinus venosus Foramen secundum Ostium of uterine tube
17
18 E i s not corr ect. 11 % c hose this.
19
Surgica l closu re of the defect is unnecessary unless the child is sym ptomatic or has evidence of a high rate of shunt flow.
Symptoms include exercise intolerance, fatigue, heart failure, or paradoxic embolization . For surgica l closu re, the ASD is
20
closed with a pericardia! or Dacron patch . Intraoperative transesophageal echocardiography is used to evaluate residual f low
21 across the surg ical site.
22 Polyethylene terephthalate Echocardiography Transesophageal echocardiogram Embolization Heart failure Pericardium Exercise intolerance Shunt (medical)

23
24

Lock
s
Suspend
0
End Block
Item: 24 of 24 ~ 1 • Mark <:] (:>- Jill ~· ~J
QID: 21551 ..1 Previous Next Lab'V!I!ues Notes Calculator

regu rg1tat1on .
2 Antibiotics Endocarditis Mitral insufficiency Infective endocarditis Preventive healthcare Antibiotic prophylaxis Regurgitation (circulation)

3
B is not correct. 20/o chose this .
4
The child does not have signs or symptoms of an arrhythmia such as pa lpitations, irregular heart rate, or syncope, and does
5 not need Holter monitoring .
Syncope (medicine) Cardiac arrhythmia Palpitations Heart rate
6
7 D is not correct. 150/o chose this.
8 Percutaneous closure of the defect is only used in symptomatic patients with an ostium secundum ASD. It is not an
9 appropriate treat ment for ostium primum or sinus venosus ASDs .
Primary interatrial foramen Sinus venosus Foramen secundum Ostium of uterine tube
10
11 E is not correct. 11 % chos e this .
12 Surgica l closu re of the defect is unnecessary unless the child is symptomatic or has evidence of a high rate of shunt flow.
13
Symptoms include exercise intolerance, fatigue, heart failure, or paradoxic embolization . For surgica l closu re, the ASD is
closed with a pericardia! or Dacron patch . Intraoperative transesophageal echocardiography is used to evaluate residual f low
14
across the surgical site.
15 Polyethylene terephthalate Echocardiography Transesophageal echocardiogram Embolization Heart failure Pericardium Exercise intolerance Shunt (medical)

16
17
Bottom line :
18
Most small atrial septal defects <8 mm in diameter close spontaneously with in the first 2 years of life.
19 Atrium (heart)
20
21
22 Refere n ces:
23 FA Step 2 CK 9th ed pp 371-378
FA Step 2 CK 8th ed pp 355-360
24

Lock
s
Suspend
0
End Block

Вам также может понравиться